SSAT AANNDD -9* PPRRAA PPRRE NN CCTT EPPAARRAATTIIOO IICCEE WWOORRKKBBOOOOKK & TEN DAYS TO THE SAT-9 GLENCOE LANGUAGE ARTS GRADE 10 Test-Taking Strategies to Help Students Succeed!

*Stanford Achievement Test, 9th edition Copyright © by The McGraw-Hill Companies, Inc. This booklet was written by The Princeton Review, the nation’s leader in test preparation. nation’s the by The Princeton Review, This booklet was written helps millions of students everyThe Princeton Review year prepare for standardized McGraw-Hill, The Princeton Review kinds. Through its association with assessments of all to help students excel on the SAT-9. offers the best way

1 2 3 4 5 6 7 8 9 024 03 02 01 00 99 Printed in the United States of America ISBN 0-07-820604-9 Westerville, OH 43081 Westerville, 936 Eastwind Drive Glencoe/McGraw-Hill Send all inquiries to: retrieval system, without prior written permission of the publisher. Act of 1976, no part of this publication may be reproduced or distributed in any form or by any means, or stored in a database Act of 1976, no part of this publication may be reproduced or distributed in any form Copyright © 2000 by the McGraw-Hill Companies, Inc. All rights reserved. Except as permitted under the United States Copyright Glencoe/McGraw-Hill Copyright © by The McGraw-Hill Companies, Inc. Preparation andPractice Introduction Practice Test Ten Days Language SA Study Skills Language Spelling Reading Comprehension Language SA Study Skills Language Spelling Reading Comprehension Reading Vocabulary Reading Vocabulary . . Table ofContents ...... 3 47 23 89 84 72 64 56 49 21 19 17 15 13 11 9 5 Copyright © by The McGraw-Hill Companies, Inc. Copyright © by The McGraw-Hill Companies, Inc. is andalways askyour teacher you when don’t understandsomething. tips, Pay closeattention to thetest-taking you’ll have into toit. putsomeeffort though, improve your score, To really gettingabetter score. your nothurt certainly chancesCompleting of thisbookletwill Will IgetahigherscoreontheSAT-9 ifIcompletethis booklet? you belesslikely will to make careless mistakes. thetest, you are ontheday notnervous of If you have practiced takingthetest beforehand. thetest if ontheday of are lesslikely to “freak out” questionsyou’ll You seeonthereal SAT-9. of acquaint you thetypes bookletwill This with itwill. Yes, Will thisbookletreallyhelpme? andstudy thetest-taking tipsandstrategies. take thePractice Test, exercises, complete each setof To you’ll developed specificallyfortheSAT-9. strategies doaswell needto asyou canontheSAT-9, andtest-taking afull-length Abbreviated Practice Test, to complete throughout theschool year, It includespractice exercisesThis bookletisastep-by-step to tool prepare designed you fortheSAT-9. What isthisbooklet? for doingyour best. andyou somesimpletest-taking canpractice techniques andtips seeonthetest, questions you will of you thetypes canreview However, Cramming fortheSAT-9 nothelpyou score. will getahigher Can IstudyfortheSAT-9? thebetter you score. will themore you practice fortheSAT-9, anything elseinlife, TheSAT-9 Like assesseshow well you canutilize theskillsyou’ve definitely not. inclass. learned No, Does theSAT-9 measurehowsmartIam? thetest. of portion helpful guideto each It isaninteractive, This bookletwascreated to prepare you fortheSAT-9. Why doIhavetoreadthisbooklet? Students from over all take theSAT-9. thecountry inschool sofar. learned The Stanford theSAT-9) 9(alsocalled to isanassessmenttest you findoutwhat designed have What istheStanford9? Introduction totheStanford9 5 Copyright © by The McGraw-Hill Companies, Inc. The SAT-9 is a timed The SAT-9 You will if learn more You one only study you You can often figure out the meaning of figure can often You a 6 This is your chance to confront the SAT-9 without the the SAT-9 confront to chance This is your Ask your teacher if teacher your Ask an answer understand why don’t you When you come across words you don’t understand, don’t you words across come you When look Whenever you hear a word that is new to you, that is new to hear a word you Whenever jot it down and Eat a good breakfast everyEat a good breakfast morning, brain doesn’t because your This booklet includes extra practice. weakest Think about your If a question, to the answer know don’t you many as eliminate Complete the practice exercises throughout the year, throughout exercises practice the Complete the complete and sections that give you the most trouble. the most you give and sections that Get comfortable using to keep track keep ofto working. you’re fast or slow how

Practice goodPractice study habits. on an empty stomach. grow to like Also, mind your keep to study short when you breaks take refreshed. try for the whole at once. test prepare to Don’t Eliminate wrongEliminate answers. is wrong. them up in a dictionary. Carry with cards note you. look it up. and vocabulary. comprehension emphasis on reading has a strong This test parts out words. figure to using word Practice on its root, concentrating by word prefix, or suffix. can, as you answers then guess! Generally, a question blank. leave guess than to to it is better section of at a time. the test on the parts Focus ofmost trouble. the that cause you the test Don’t be afraid ask questions. to Don’t you. books that challenge Read pressure ofpressure and weaknesses strengths test-taking your evaluate test. be able to the real You’ll of day the before habits your and change test. the real mistakes. Learn your from in advance,areas weaknesses. your allot extra time for strengthening study to sure and make Wear a watch Wear seriously. Test the Practice Take Practice Test under simulated test conditions, test under simulated Test Practice tips and techniques. learn and test-taking the the questionsPractice difficult questions. answer to techniques test-taking under timed conditions. exercises some SAT-9 completing Practice test, of although versions different time limits. different have the test Complete this booklet. this Complete • • • • • • • • • • • • •

Stanford 9 Introduction

As you practice for the SAT-9 throughout the school year, try the following: to do each of year, the school throughout for the SAT-9 As you practice Preparing for the SAT-9 for the Preparing Copyright © by The McGraw-Hill Companies, Inc. Look attheunderlinedword.Doyouknowwhatitmeans? Ifnot,here’s whatyoushoulddo: For example,let’s usePOEtofindtheanswerquestionbelow. In themeantime,keepthesefivetipsinmindasyoucompletepracticeexercises: We work through each will technique indetail helpyou and techniquesThere thatwill to strategies are doyour several important best ontheSAT-9. Five Tips forCompletingthePracticeExercises r r r ? 1. a.Ee fyucnol ueotoeo w nwr,yu hne fguessing correctly increase. will your chances of you outoneortwo canonlyrule answers, Even if can. asmany answers wrong asyou of to getrid you shouldtry In thesecases, question. (POE). Elimination Process of Tip No.3:Eliminatewronganswer choicesfirst the firstchoices iscorrect! Sometimesyou findabetter answer will asyou read on. you thinkoneof even if theanswer choices, to read of It important all isespecially answer.wrong you may misunderstandaquestionoranswer choice andpicka you read carelessly, If Tip No.2:Readthequestions andanswerchoicesslowly each test section. of them to getthe andfeel” “look Just review Don’t answer onthesamplequestions. waste timechoosing theright test. the appearbefore each asamplewill newsectionof Onthereal test, thetest. of part Sample Questions. Always make sure you understandthedirections anewsection. before starting the test. questionthatappearson each sectionof directions tell you of how to answer thetype Directions. Tip No.1:PayattentiontotheDirectionsandSamples D C B A Something thatisext hs od nmn syulo talo theanswer choices. those words inmindasyou lookatallof from thewords you may know theprefix “ex-” In thiscase, words. other theword thatyou of candefineorrecognize asparts there are any of parts Decide if irritating thorough defensive tiresome edtedrcin aeul hnyubgnec etscin The Read thedirections carefully whenyou each test begin section. and carefully The samplequestionsshow lookoneach how thequestionswill e nsi v e is — oeie,you notknow theanswer will to a Sometimes, Ten DaystotheSAT-9 7 . extra or extend .Keep Copyright © by The McGraw-Hill Companies, Inc. ? ? and and extend extend ends extra extra or or extra extra defensive . If you had not been . have anything in common with in common anything have distractors have anything in common with common in anything have extensive have anything in common with in common anything have have anything in common with in common anything have irritating 8 search for something,search looked it means that you’ve thorough tiresome defensive . (B). answer Eliminate thorough means “annoying.”means Get rid of this answer. protective means means “boring.” means rid Get of answer. this Irritating ? No. ? Yes! IfYes! ? do a you Tiresome Defensive careful in the question above, you might have been tricked into careful in the question above, you might have been choosing wrong answer (B) just because the word in “-ensive” just like the word Some wrong answer choices are included just to confuse you! Some wrong answer choices are included just to These answer choices are called

extend No. extend about part know of thinking about what you By the underlined word, determined you’ve choice. answer best (C) is your that answer Now look at answer choice (D). choice look at answer Now Does Look at answer choice (B). choice Look at answer Does (C). choice answer Look at Does “extra”in everywhere—even thought look. to not have other people may places Keep (C). choice answer Look at answer choice (A). choice at answer Look Does No. Try not to read a question more than twice. a question more read not to Try Read each question once slowly and carefully to make sure that you understand it. understand that you sure make to carefully and slowly question once each Read wrong. are know you answers eliminate to Try If one answer, only eliminate can you take on. if it later to back best guess and move come your can always You time. have you Think only about the question you are working on at any given time. given on at any working are Think only about the question you Watch out for distractors! Watch Don’t be fooled!

• • • The SAT-9 is a timed test,The SAT-9 yourself. pace to have so you of as many answer to Try the can without being careless. questions as you yourself: pace help you tips to some are Here • You will for guessing on the SAT-9. not be penalized You wrong as many Eliminate can, as you answers guess. then if Even answers, any cannot eliminate you still you of a 25% chance have guessing the right answer. time on one question No. 5: Don’t spend too much Tip Tip No. 4: If you don’t know an answer, guess! answer, No. 4: If you don’t know an Tip 6. 5. 4. 3. 2.

r

r Stanford 9 Introduction Copyright © by The McGraw-Hill Companies, Inc. Preparation

Preparation Bzzz Practice Practice and and 9 Copyright © by The McGraw-Hill Companies, Inc. Copyright © by The McGraw-Hill Companies, Inc. 5 4 3 2 1 same astheunderlinedword. theword thatmeansthe Mark theanswer of DIRECTIONS To To D C B A fsomethingis If J H G F To To J H G F To D C B A A D C B A Reading Vocabulary shr pacify d ap tasteful fair clever knowledgeable crave degrade smell humiliate analyze enlarge arrange soothe typical difficult reasonable remote estimate horrify praise appeal e e g pal w e ne d l is to — is to — person is— person r at e is to — g r ue ling itis— , 11 6 7 as inthesentence. underlined word isusedinthesameway thesentence inwhich the answer of Mark the Read thesentence inthebox. DIRECTIONS B In which of the following doesfuse thefollowing In of which D F above? mean thesamethingasinsentence In which of the following doesmat thefollowing In of which A the samethingasinsentence above? G J H C tie to mat tie takeIt meanhourto finda will id fmusic. kinds of attempted to fuse thecomposer In thenewpiece, replace the we hadto When went out, thelights Chinese cooking styles. The scientistlitthe easily. I lighted a I lighted Tonight’s dinnerwill experimental rocket.experimental I saw abeltthatwill My aperfect andIform wife I’ll perfectly. He hasashort mat c h c h wits with himany with time. wits mat this shirt. fuse c fuse . h in thedarkness. two different sohegetsangry , fuse mat fuse on the c h GO ON GO French and your shoes mat c mean c B h h A . C Copyright © by The McGraw-Hill Companies, Inc. . r o ,so le at y r le STOP o mean? mean? y nsab nspir r e n o o nsab dilat c e nig e dilat and caused b n mean? r nig o e b at nspir o c Partner Strategy Transportation Automobile Devastating Mild Hot Secret Timeless Important Elegant Sensible Hardworking Fearful Careful Tardy A B C D little damage. does What Sue knew that Carl was often Sue A B C D very listened the because carefully Inez instructions provided indisp What does What information. does indisp What mean? F G H J worked the burglar not have could Since alone, for his searched police she didn’t fret about being on time about being fret for she didn’t their meeting. does What F G H J The storm proved DIRECTIONS sentence. each Read question, each For mark of the answer means the same, that the word the same,or almost word. as the underlined 12 13 10 11 12 n o the mean n . o n o in the on the entire the ship. at the river. at n n aband o o stand stand stand aband aband . n o of trees. Saturday afternoon. Saturday a The soldiers made idea. If is a fire, there must we building. The sailors had to The sailors had to aband lunch line. lunch set up a lemonade Jay stand The rock star played the guitar with star played The rock We were forced to We Every day,Every the children We stopped to rest in the shade of in the shade rest to stopped We a A palace guard must stand must guard A palace attention without moving. The dancers moved with aband moved The dancers J D A mean the same thing as in the sentence above? H In which ofIn the following stand does B C G In which ofIn the following does aband F the same thing as in the sentence above? in the sentence the same thing as

C

A B 9 8 Stanford 9 Preparation & Practice Copyright © by The McGraw-Hill Companies, Inc. question. Mark thebestanswer foreach Read thepassagebefore reading thequestionsaboutpassage. DIRECTIONS Reading Comprehension omne,andIexpect to seethegreat metropolis replaced scale.” onamuch grander commenced, rebuilding SanFrancisco has work of “The 1906, announcedCalifornia on April 23, Thegovernor of thefire was contained anddoused. hours, after almost100horrific Finally, to make theirway streets to through thecrumbled tackle theblazes. SanFrancisco wasengulfedinflamesbefore firefighters could begin rate thatmostof alarming Thefire spread atsuch an thecity. especially inthetenement of housing inthepoorer parts many, Thisfire took thelives of A fire calledtheGreat Fire resulted from theearthquake. 20,000refugees to safety.carrying the thecrew of addition, In Soldiersalsoorganized andbuilttent citiesforrefugees. discourage scoundrels from stealing. order to Themayor even issueda “shoot to kill” others seekingto profit from thechaos. Troops arrived from to across protect anditsresidents thecountry from thecity looters and United States Army to restore thefires order throughout andhelpfight thecity. SanFrancisco uponthe called Themayor of thathadremainedbuildings to intact crumble. It alsocausedthefew theinitialquake. ordeathduring residents whohadescapedinjury havoc wreaked This further onthe majoraftershock a struck. Three hoursafter thefirstshock, communications were cutoff. telephone andall andtelegraph power, lostall city The thatblazed forfourdays. inferno raging a andstarted to causedbuildings collapse, more than 3,000people, disaster took thelives of natural Thishorrible 1906. theGreat of Earthquake themany heroes of Fox isjustoneof onechild being injured thiswithout orlost. Sheaccomplished of all San Francisco Bay. Fox thattook across themto safety the gathered thechildren andputthemonaferry quickly, Thinking Sheeven clothingforthechildren salvaged enough to wear. to geteveryone outside. thehouseandmanaged movedThey thechildren through thecompletely demolishedinteriorof charges. andprotecting comforting their from ran room andthey to room, She assembledherstaff Fox flewinto action. SanFrancisco. 1906hadjustrocked of thecity The Great Earthquake of thechildren screaming. andthesoundsof shaking, earth smashing, glass caving inaround her, Fox’s dreams were quite literally shattered at5:13 closed hereyes andfellinto apeaceful sleep. She Nighttime wastheonly timesheexperienced quiet. theplace. reveled inthequiethush of Fox thecovers pulled upto herchin and Maria Kip wasover Orphanage waswell. andall Another day atthe andwent to bed. theshades, of closedall theorphanage, locked thedoorsof Alice Fox inhercare, to the125orphans saidgoodnight 1906, April 17, On theevening of USS Chicago h ihyFre fNature ForcesThe Mighty of accomplished one of the largest evacuations by thelargestevacuations sea, accomplished oneof 13 A . M h etmrig Sheawoke to walls thenext morning. . GO ON GO Copyright © by The McGraw-Hill Companies, Inc. STOP ? USS Chicago means — A city A ship ofA troop soldiers A train 1 4 6 8 money children teachers victims She hid them under the cellar stairs. cellar them under the She hid the windows to them from She threw waiting firefighters. the water. across She sailed them city. a tent them to She evacuated 12 hours 24 hours 72 hours 100 hours A B C D paragraph to read Which should you learn what took actions the government the earthquake to and fire? in response F G H J paragraph, the second In the word charges A B C D How did Alice Fox secure the safety secure of Fox Alice did How in her care? the children A B C D long did the Great how Approximately last? Fire F G H J was the What 7 8 9 5 6 14 — except Ambivalent Envious Respectful Fearful To tell readers all about her life and all about her readers tell To work a historical event personalize To dangers of the expose To orphanage life at the turn of century the is an earthquake how explain To caused Alice Fox saved more lives than any lives more saved Fox Alice ofother hero the earthquake ever was the largest fire Fire the Great shoot people in to forced were troops looting prevent to order areas lost in wealthy were lives fewer of the city than in poor areas the arrival of other states from troops issue ofthe mayor’s kill” to “shoot a order the organization of a soup kitchen the building of city a tent The passage mentions all of the to in response following actions taken the earthquake F G H J A attitude toward the writer’s is What Fox? Alice Why does the author include the story does the author Why about Alice Fox? B C D F G H J From the informationFrom in this passage, that — can conclude we A B C D

4 3 2 1 Stanford 9 Preparation & Practice Copyright © by The McGraw-Hill Companies, Inc. 4 3 2 1 itk,choose mistake, you findno If the sentence themistake. with theanswer of mark you discover amistake, If underlined word bespelledincorrectly. will It ispossiblethatan Read each sentence. DIRECTIONS A J F D G G F D J C C A H B B H George waselected to the student No mistake They wereThey No mistake Will managedto The dishwascovered with directions. The group was No mistake No mistake Iris emergedfromIris theaccident thescene was After thefire, unscathe se c micr p Quincy took thecomplaints r Suarez Mr. thestudentsliked theway that All of land. the smallplotof an olwdteplce fhis Manny followed thepoliciesof Ursula always incomplete writes The ounse r nt e d e e o p nc c o r eso l o rg es . fit No mistake d Spelling anisms . r . s saw into thefuture. . assid tau t . uous s the class. uspicious ek . e in following the in following out a living on out aliving foutsiders. of c se haot r iousl ic . y . 15 6 5 9 8 7 A B H G C G F D D A J A J H B C C F D B The audience admired the John tore apagefrom the involved inthedecision. was justa but it We were there atthesametime, I am r Stanton beganto collect a Mr. c thepoemwas The mannerof about hergrandmother. told anamusing Carrie No mistake when heretired. d No mistake c hist There were three No mistake The illnesslefthim No mistake The sunlight passedthrough a The sunlight We searched the The The concerned a citizens formed information. etidt amhm but hewas We to calmhim, tried No mistake Nancy’s movements were slow and Kate’s speech. o o e n mmitt lib v e r e v e ical inc r r il sat at lain e line e . io e . . c nal wore ablackhatandcape. o d incid to agree with Harry.to with agree . ar e dist nc ci w v e inc es . r a c for the t ane k factors GO ON GO e cale d r c e d with pain. with t ot o nd r p p e ic e r

e B

nsio z ism

r z

in z . . n Copyright © by The McGraw-Hill Companies, Inc. . s . r e of d ear l p ar o STOP ttle b dical nne ue a disap r me so when we when . r r e hniq p c ious e t v heale to the left. to b d o comes everycomes , should ask you . e skue nt ic um v d lived near the castle. near lived me a nni e v le . e mo . mil nc e easants p lig uine e n e saw all the information. saw A new the scouts. for a the trees examined The hunters sign of The magician made a rabbit Lester was impressed with was impressed great her Lester The thousand years. mistake No improved. was The picture changes. mistake No the details of kept He his himself. life to was The conclusion g int mistake No Mrs. to rooms rented Clanton No mistake mistake No If need you The scholar said that the coin was the coin said that The scholar The ten-mile hike tested the tested hike The ten-mile No mistake mistake No practiced, Kelly As her The new manager suggested Cleon. Dr. is a great Andrews C C A H C D G J A B J A D F B B D F H G 17 18 19 15 16 16 . nd d e r p on ie y . e st w y l nc this morning. nal iole about her v s y of profession. her casio early about his work. r from the starting from c o y le o d c st e ing ntl at it . r ue order. le q ns of the story was a pinna e c lo io the box this morning. the box c e ical a ine g tat novels are my favorites. my are novels o . e nd r r y e ir e r p e he p nolo o r st o mo e y hr line. The The fish were b The fish were No mistake No No mistake No The group objected to to objected The group vacation. Sal visits his sister c His actions were open to many to open actions were His int M me The student received a small received The student for his efforts. The cars He spoke spoke He television. mistake No After years of years After work, hard Clara the reached nine-year-old girl. nine-year-old mistake No The museum showed the paintings in showed The museum We I thought the movie too was He wrote about his childhood in his about his childhood wrote He Sarah told funny funny Sarah told No mistake No

H A J J C G B F G F D A J G H C H B F D

z

z z

Stanford 9 Preparation & Practice

14

13 12

11 10 B Copyright © by The McGraw-Hill Companies, Inc. 4 3 2 1 itk,choose mistake, you findno If best corrects themistake. choose theanswer that discover amistake, you If these. oracombination of word usage, capitalization, mistake may be inpunctuation, The thesentence. of in theunderlinedportion beamistake There might Read each sentence. DIRECTIONS D C B A thegroup. the backof I prefer D C B A to believe. J H G F winners. De On ac R J H G F ic wa har Correct asis Because hewassotallJim Jim Because hewassotall, Jim hewassotall On account of Correct asis Richard’s explanations are Richards explanations were Richard’s explanation’s are Correct asis were Dewayne andTonya, andTonya were Dewayne, Dewayne andTonya was Correct asis classical music classical Music Classical music yne andT c ds explanat ount o C lassical M Language Correct asis f hewassotal o n ya w io usic ns ar e r e to any otherkind. . e among the always difficult l, J im stood in 17 6 5 7 Correct asis or mark best way to thesentence, rewrite the Mark theanswer of sentence structure. Study thesentence inthebox foranerror in DIRECTIONS J H G F D C B A than ad The space shuttle D C B A the a He and speech madeashort finished the report eventually.finished thereport We worked foralongtime, Correct asis continues old, Atlantis old continues Atlantis old continues Atlantis Correct asis after we worked foralongtime. eventuallyThe report we finished we worked foralong time. Eventually we after finishedthereport eventually finishedthereport. We worked foralongtimeand Correct asis accepted theaward humbly. accepts theaward humbly. accepts theaward humble. war e fthesentence iscorrect. if d h cad umb which ismore thanadecade , which ismore thanadecade , which ismore thanadecade e old, le c . A o tlant nt in ues is w ob fuse. of to be hic GO ON GO ac h ismo c e p t e d r e Copyright © by The McGraw-Hill Companies, Inc. STOP word to replace the replace to word best Keisha when she finished her dance, when she finished Keisha cheered. everyonein the room finished her dance, Keisha When cheered. everyonein the room finished her dance,Keisha in everyone cheered. the room as is Correct handsome exactly pointed witty Keisha, in the room everyone cheered, when finished her dance. she Which is the Which A B C D underlined word in the paragraph?underlined word A B C D , so it is important be on time. to The trip p 9 11 shar . M . 18 P .on . the by .on Paragraph 1 M . M M P . . P P . the school. entrance to the front Friday. meet them by on We’ll M way to way . P best . Friday. on M . Friday. is the school entrance to the front By at 5:00 meet the group we’ll where P front entrance to the school Friday. the school entrance to front at 5:00 meet the group We’ll the front meet them by we’ll Friday the school. entrance to the front by group meet the We’ll at 5:00 the school entrance to We’ll meet the group at 5:00 meet the group We’ll She plants the flowers in the the flowers She plants backyard, them every watered day. in the flowers the She planted them every day. and watered backyard in the the flowers She was planting backyard, them every and day. watered as is Correct She planted the flowers in the the flowers She planted backyard, them every watered day. D Which ofWhich these is the G H F combine the first two sentences? the first two combine F G H J

We’ll meet the group at 5:00 meet the group We’ll The bus to Washington, The bus to D.C. at 5:30 will leave will take about three hours, about three will take should bring eat on the bus. a snack to so you report The weather Washington, it will that in predicts be cold D.C. this weekend; bring to warm clothing. be sure DIRECTIONS the paragraphRead and the questions that follow. question. for each the best answer Mark 8

10 Stanford 9 Preparation & Practice Copyright © by The McGraw-Hill Companies, Inc. 4 3 2 1 Read each thebestanswer. questionandmark DIRECTIONS be would the following Knowing of which J H G F ahardware store? contain theaddress of J H G F Angeles? Los of thecity about articles magazine What isthebestplace to lookfor D C B A library’s catalogsystem? Which of the following is thefollowing Which of D C B A would use— you text, alineof word attheendof To way findtheproper to hyphenate a least An atlas A telephone directory A newspaper A dictionary Literature Guide toPeriodical The Readers’ An atlas inPrint Books An almanac thebook The titleof thebook The lengthof thebook The subjectof thebook theauthorof The nameof an almanac an encyclopedia a thesaurus a dictionary useful infindingabook Study Skills most likely to 19 Contents andIndex to 7–11. answer questions Use theTable of Index from ascience book. Contents and istheTableThe following of 6 5 Chapter 5 Chapter 4 Chapter 3 Chapter 2 Chapter 1 J H G F on thispage? thesewords would befound Which of dictionary. Below are guidewords from apagein D C B A on thispage? thesewords would befound Which of dictionary. Below are guidewords from apagein ula ea .105 Nuclear Decay . .78 Acids .61 Liquids .36 Gases .3 Atoms sandal salivate salve sandbar piston placid pitiless placate CONTENTS salute —sample pity —placelesspity GO ON GO Copyright © by The McGraw-Hill Companies, Inc. not STOP not an acid not Boyle’s Law Boyle’s Charles’ Law Law Dalton’s Law Hund’s Page 6 Page 13 21 Page Page 28 2 1 and Chapter Chapter 3 2 and Chapter Chapter 4 3 and Chapter Chapter 5 4 and Chapter Chapter Arrhenius Bohr Bronsted-Lowry Lewis Chapter 1 Chapter 2 Chapter 3 Chapter 4 Chapter deal with gases? F G H J ofWhich following the is contain informationcontain about the periodic table? F G H J contain chapters two Which information about boiling point? A B C D ofWhich does the following laws definition? A B C D In which chapter would you find you would which chapter In information about protons? A B C D ofWhich the following pages does 9 7 8 10 11 20 80 30 41 111 69 11 10 6, 13, 28 INDEX 58 40 108 47, 68 45 21 79 65 66 4, 14Ð22 8Ð11 20Ð21 100 5Ð9 6

114

molarity molality Lewis 82 Lewis Bronsted-Lowry Arrhenius alpha decay alpha decay beta decay concentration, Boyle’s Law Law Boyle’s boiling point buffers Bohr atomic radius atomic radius acid, of, definition valence valence quantum theory Dalton’s Law Law Dalton’s periodic table proton Charles’ Law electron freezing point Law Graham’s Law Hund’s mass number neutron decay,nuclear types of, Stanford 9 Preparation & Practice Copyright © by The McGraw-Hill Companies, Inc. 6 5 4 3 1 misspelled. If there is no mistake, mark mark there isnomistake, If misspelled. theunderlinedword thatis theanswer mark choice of there isamistake, If words ismisspelled. theunderlined It ispossiblethatoneof Read each sentence andlookattheunderlinedwords. DIRECTIONS edec asg n h usin htflo.Mark thebestanswer foreach question. Read each passageandthequestionsthatfollow. DIRECTIONS Petra was The We We Tim’s gave an story D C B A Taylor Coleridge? Samuel about recent articles magazine Where shouldSimon lookto findany v A dailynewspaper A dictionary An encyclopedia Literature The s isit ub BCD AB G J FGH Language SA e je edr’Guide toPeriodical Readers’ d ct amaz the fthe of FGHJ t r e o d omsato,Samuel Taylorpoem’s Coleridge. author, aboutthe sohehasdecidedto write favorite poem, about apoet. Simon’s areport classisto write forEnglish assignment pical when shesawwhen the ar altar t ical BCD C AB rain rain e was an d Samuel Taylor Coleridge v f e o r r h ieo the Ancient Mariner The Rime of sio r est ar n . r fthe of No mistake N o sp g o mistak ant e ctacular person. e v 21 e nts e . . . paint 2 N N o mistak o mistak J H G F include inhisreport? to item forSimon to try important hc fthesewould bethe Which of ing . The time when Coleridge livedThe timewhenColeridge likedThe foodsColeridge to eat poems Coleridge’s someof The titlesof The place where lived Coleridge N e o mistak . e . is Simon’s e . GO ON GO least SA SA

A rough draft of Simon’s report is below. Use the rough draft to answer questions 7 and 8. The Life of Samuel Taylor Coleridge Samuel Taylor Coleridge was one of the greatest poets in the (1) English language. He wrote during the Romantic period of English (2) literature, which took place in the late 1700s and early 1800s.

His most famous poems are The Rime of the Ancient Mariner and (3) Kubla Khan.

Samuel Taylor Coleridge was born in England in 1772. He was an (4) (5) imaginative child who kept to himself. Rather than play with other (6) children, he preferred books. He began writing poetry as a (7) teenager. A sonnet is a poem that is fourteen lines long. He went (8) (9) to college in Cambridge.

Coleridge did his best work when he was a young man. His (10) (11) greatest poems were all written in one year, when he was only

twenty-six years old. During this “wonder year,” he wrote The (12) Rime of the Ancient Mariner, Christabel, and Kubla Khan. These Copyright © by The McGraw-Hill Companies, Inc. (13) three poems, are among the best ever written in English.

Coleridge continued to write fine poetry in later years, but (14) it was not the equal of his earlier work. He died in the summer (15) of 1834.

7 Which sentence does not fit in the report? 8 Which of the following would be the best way to begin sentence 11? A 5 B 8 F Nevertheless, C 10 G Despite this, D 13 H In fact, J But still,

Stanford 9 Preparation & Practice 22 STOP Practice Test

Pay attention to the directions and sample questions. Read the questions and answer choices slowly and carefully. Eliminate wrong answer choices first. If you don’t know an answer, guess! Don’t spend too much time on one question.

23 Copyright © by The McGraw-Hill Companies, Inc. GO ON is — nt ale v is to — is to e is to — is to a thing is to — a thing is to ulat ume means — means — is related to — to is related y base es cum e g ful education agriculture religion law worry witness make-believe conclude kind-hearted rude sorrowful excited needy aloof conflicted respected mimic gather waste frighten mistake it mistake corrupt it it endure mention it r le ue F G H J To d F G H J A person who is ambi A B C D C R A B C D To ac F G H J A B C D To pr Reading Vocabulary Reading 8 9 5 6 7 10 24 is — le is — nt e d is to — is to is to — is to is to — is to e e ound tat r u nit ur circle enter peel straighten surprise understand modify supply start repress soak purchase amused agreeable just practical forceful forceful independent flirtatious joyful To m A B C D A person who is ar A B C D F G H J To ig A B C D To s A person who is amicab F G H J

C

SAMPLE A DIRECTIONS of answer the Mark that or words the word same,means the almost the same, or as the word. underlined B 4 3 2 1 Stanford 9 Practice Test Copyright © by The McGraw-Hill Companies, Inc. 11 as inthesentence. underlined word isusedinthesameway thesentence inwhich the answer of Mark the Read thesentence inthebox. DIRECTIONS SAMPLE the samethingasinsentence above? the samethingasinsentence above? C sohe John wasthemostorganized, D A doespr thefollowing In of which A doesthr thefollowing In of which B C D B I worked onthepr The authorwasableto thr several subplotsinto hernovels. The sheets are made with softthr The sheetsare madewith andfantasyintoreality hispaintings. thep was madeheadof a largeprofit next month. An actor must p fsuch alongplay. of The tailorcould notthr We canp ae norrsac,we canp Based onourresearch, theauditorium. wall of It istoo difficultto follow thethr too hardThe painter tried to thr r o je c t o the filmonfront r je o ct je c for months. t his voice. r ea o e je d ad c the needle. t o . e je ad ct r ea o ea ea mean mean je d d d c . t 25 13 12 14 J B F the sentence above? C G D H A the samethingasinsentence above? F in thesentence above? G c does thefollowing In of which c does thefollowing In of which In which of the following doesse thefollowing In of which J H o o container stays sothefood fresh. too much to work. time ypoya h g ffifteen. symphony attheageof aey hehasbeenc Lately, Make sure to se to sure Make The pianistc mmitt mp The cook c day.parents every The partners decidedto seal The partners The woman remained c The trainer taught theseal taught The trainer ahandshake.with The rock isc when thepolice told her thenews. president’s seal thedocumentdoesn’t have the If recognize hersignals. The boy c I forgotto seal C c Sometimes peoplehave ahard time center probably saved hislife. after realizing thedinnerburned. c Margaret’s parents are usedto c The environmentalists hadnotrouble o o o o ose mmitt mmitt mmitt mmitt ing d mean thesamethingasin ing ing ing mean thesamethingas ing o o mp o mp funds to charity. to theircause. to arelationship. o mp him to thetreatment mp al ose itisn’t valid. , the envelopes. ose ose the plastic ose d d o a letter to his d mmitt d isl quickly himself her first fquartz. of o mp GO ON GO ing to ose the deal al d mean B A C Copyright © by The McGraw-Hill Companies, Inc. STOP mean? amounts of . does What ic t e r pious pious o o mean? and needed to be more and needed to ious when he refused to wear when to refused he nt e t mean? e ious ic nt mean? t e Snobby Cheap Worldly Helpful Elevated Expensive Colorful Crude Nonbeliever Nonbeliever Woman Criminal Judge Minimal Abundant Unnecessary Contemptible e t r e A B C D Mr. artist’s thought the young Purdue was base work F G H J A B C D of weeks After rainfall, heavy the town was flooded with c F G H J Maggie thought her friend was being pr he clothes from unknownclothes from designers. What does pr sophisticated and refined. does What base water. does c What Because she denied popular beliefs, she denied popular Because Joan of was called a he Arc 19 20 17 18 26 . ip mean from an from ip mean? ip d e hid the teacup as she the teacup ip her son for eating when I spoke to him, to when I spoke d t e e e hid mean? t of sharp. pencil is too my e e of tree. the ip ip Congratulated Loved Scolded Tempted The detective received the t received The detective walked across the crowded room. the crowded across walked The t anonymous source. anonymous Beth tried t not to The serviceso good that Mr. was a hefty t leave decided to Garcia Rude Noisy Mobile Careful He found the eagle’s nest at the the eagle’s found He very t

Mrs. Carlton c I had to be discr I had to A B C D too quickly.too c does What In which ofIn the following does t B D C A the same thing as in the sentence above? in the sentence the same thing as F G H J so I waited until we were alone. until were so I waited we What does discr

C

SAMPLE

A DIRECTIONS sentence. each Read question, each For mark ofthe answer that means the same, the word or almost the same, as the underlined word. B Stanford 9 Practice Test

16 15 Copyright © by The McGraw-Hill Companies, Inc. question. Mark thebestanswer foreach Read each passagebefore reading thequestionsaboutpassage. DIRECTIONS Reading Comprehension SAMPLE D C B A Sergei casthisfishinglineto — show hisfrustration throw away thefishingline make room ontheboat to catch fish try enough fishto cookenough afeast. He would sooncatch found medium-sized walleye onthehooks. pulledinthelinesand Sergei to bite. Thefishstarted some distance. could Sergei seefor brokelight through andtheentire skywaslitup. suddenthemoon’s Thenallof but itwascovered behindtheclouds. Themoonhadrisen, theboat. Sergei casthisfishinglineover thesideof nEeigo Fishing An Evening of 27 I.Sergei pullsupfishinglines III. .Themoonlightisblocked by clouds I. II. J H G F line II? on belongs thefollowing Which of passage. the The outlinebelow describes Sergei findsmedium-sized walleye. He would soonhave fish. enough fishingline. a Sergei castsoff to bite.The fishstarted GO ON GO Copyright © by The McGraw-Hill Companies, Inc. ,at GO ON . . Roots in 1972, the first Afro-Americans in the Afro-Americans Singin’ and Swingin’ and , On the Pulse ofOn the Pulse Morning Georgia Georgia Phenomenal Woman , and , was published in 1970 and was a huge 28 . of She also taught at the University Ghana. Gather Together in My Name in My Together Gather And Still I Rise And , The Arab ObserverThe Arab AfricanAmericans and Women Maya Angelou:Maya Ground for Breaker I Know Why the Caged Bird Sings the Caged Bird Why I Know

. Haley’s Alex in for her performance She was highly commended Gettin’ Christmas Merry Like literary her addition to In accomplishments, Angelou, a civil rights activist, was Dr. by asked King, Martin Luther Jr. for the Southern Christian be a coordinator to in the 1960s. Conference Leadership Also, she has held the position of 1981 since of Professor Reynolds University. Forest is fluent Angelou Wake American Studies at languages,in several including Italian, French, Spanish,African Fanti. West and in the film industry,American women African for new ground Breaking Angelou has written movies, scripts and directed including “The honorary duty of love,” being is to a human Angelou. states writing Her this belief,stresses as the importance as well of steadfastness, tenacity, courage, and the importance of day. best each doing your her poetry, Through fiction, and other efforts, these lessons gently and completely. teaches Angelou age seventy-one, At she American women,African for new ground break to continues and all women everywhere. success. virtually In all of her writings, and presents women celebrates Angelou and smart, strong as female characters models for all role excellent providing women. include works Other screenplay written a black woman. by screenplay television appearances She has made many documentaries, award-winning and has produced including Arts In January 1993, January In in rapt attention, nation listened as the author, poet, actress, playwright, her poem, read Angelou and historian Maya President Bill Clinton’s inauguration. Clinton’s Bill President this moment, Before of heard some had never of the pleasure her or experienced hearing strong, her rhythmic, voice passionate ofspeak words truth. universal new ground, again broken had once Angelou as a American. African and as an woman 4,April on Johnson was bornAngelou as Marguerite 1928, her and was raised by Arkansas.grandmother in segregated pain and heartache the She experienced of racism, success.achieve to obstacles many overcame but graduating high After from in San Francisco,school California, with her mother, she moved where a she took series of jobs including waitress, different cook, actress, and singer. she Eventually Cairo, to moved of she was the editor where the only English-language newspaper in East, the Middle was one ofAngelou write to a best-selling book. American women African the first novel, Her Stanford 9 Practice Test Copyright © by The McGraw-Hill Companies, Inc. 4 3 2 1 B A Maya was— Clinton’s inauguration, When sheread her poematPresident J H G F How thisarticle? didtheauthororganize D C B A that Maya was— to emphasize attention” listened inrapt that The authorwrites “the nation J H G F the article theseopinionsistheauthorof Which of D C the editor of directing systematic arguments of Series Issues andresolutions Chronological order Initial concept followed by support a good producer a talented orator a courageous woman an excellent actress than WakeForest. Maya would teach rather atGhana Maya’s proudest moment. The Emmy Award nominationwas screenplay writer. Maya istheworld’s bestblackwoman Maya. Luther Martin King respected Dr. a professor at Wake Forest University Constitution a reporter for most likely eri,Georgia Georgia, The ArabObserver The Atlanta to support? 29 5 6 III. .Enduredsegregation inArkansas I. IV. Worked with Dr. Martin LutherKing, Martin Worked withDr. IV. I Graduated fromhighschool II. Angelou’s life. Maya of part This outlinesummarizes F the reader already knows — thispassage assumesthat The authorof D C B A What inthemissingentry? belongs J H G Jr. what is written in what iswritten black woman Wrote thefirstscreenplay by a written Acted in Moved to Cairo Produced fGhana of what attheUniversityAngelou taught is what aninauguration Angelou’s poem inaugural the text of Roots Afro-Americans inthe Arts The ArabObserver GO ON GO Copyright © by The McGraw-Hill Companies, Inc. Elk Bears GO ON Foxes Eagles Moose Snakes Buffalo Wolves Look ! Caribou Coyotes Animals to Stay Clear Of Want To Miss To Want But Don’t Touch Animals You Won’t Won’t Animals You . daily. All persons are strictly are strictly All persons daily. . P.M 30 . until 10:00 . A.M For your safety, please observe the following rules: observe please the following safety, For your

Children should be supervised at all times. The park cannot be Children should be supervised at all times. areas. accidents occurring in playground for held liable Several park ranger stations are located throughout the park. In the park. park ranger stations are located throughout Several station using maps located find the closest case of emergency, main park area. in each on placards Hiking trails are provided throughout the park for your your the park for throughout Hiking trails are provided from deviate should hikers experienced Only enjoyment. and gear is also hiking clothing Appropriate these trails. and several in places, get as brush can thick recommended, terrain. in the park create challenging gorges rocky Parking is permitted only in designated parking lots. Vehicles Vehicles designated parking lots. in is permitted only Parking parked Vehicles not be parked on grass or other areas. may be other than the designated parking lots will anywhere immediately. and towed ticketed Grills are provided areas only. Picnics should be held in picnic be Coals should outdoor cooking needs. for pavilion in each All when cooking is completed. doused with water immediately picnic site and located at each refuse MUST be placed in cans fined $500. caught littering will be Persons the park. throughout Pets are not allowed. All persons found on the premises with pets (even if pets with pets (even on the premises found All persons are not allowed. Pets and will be issued a fine of $100. promptly are on a leash) will be removed Park hours are from 6:00 are from hours Park prohibited from park areas at any other time. park areas at any from prohibited

7. 7. 6. 6. 5. 5. 2. 2. 3. 4. 1. 1. Remember: Keeping Our Parks Beautiful Is Everyone’s Responsibility!

P.M. A.M.

to Welcome to Whitestone Regional Park Whitestone to Welcome Hours 6:00 10:00 Stanford 9 Practice Test Copyright © by The McGraw-Hill Companies, Inc. 9 8 7 D C B A Park inautomobiles — Families entering Whitestone Regional J H G F meant to — are suggeststhattherules The sign D C B A Which of the following isan thefollowing Which of can enter hoursaday twenty-four area inadesignated should park are allowed leashedpets to bring children attheplayground can drop off keep safe visitors keep from visitors grilling keep hikers challenged keep vehicles safe Gorges canbechallenging. Trails are fun. Pets are nottolerated. Persons befined. wholitter will opinion ? 31 11 10 12 J H G F go? should visitors where anemergency, In theevent of D C B A littering? is caught who visitor to happen What apark will J H G F between buffaloandbearsis— suggeststhatadifferenceThe sign The firstaidpavilion Any station ranger lots parking Designated areaThe mainpark He beissueda$500fine. orshewill He from beejected thepark. orshewill His orhervehicle betowed. will He beissueda$100fine. orshewill how faraway to stay the placards usedto locate them at them the gearrecommend forlooking the park’s foraccidents liability GO ON GO Copyright © by The McGraw-Hill Companies, Inc. GO ON 32 The Last Great Race on Earth on The Last Race Great

This first race along the inspired many modern mushers to test their own test modern to mushers many inspired Trail along the Iditarod This first race endurance. year, Every “The in what is called compete dogsled teams seventy-five about did more just as Kaasen on Earth.” Race Last Great Trail They sprint the Iditarod along than half a century ago. of the courage can match no one But of teams those first brave tragedy. from town a tiny save to their lives men and dogs who risked Quickly, the final grueling and mushed Nome. his team miles into he reassembled With the serum on hand, disaster. from was protected the town Alaskan men The fearless 127 hours. journeyand their dogs made this incredible in just over was record- It time. breaking As ,As lead dog, Kaasen’s the raging on through forged winter storm, burst of a huge rag dolls. them down like the air and threw into wind the sled and dog team lifted “The serum is gone!” fix his sled and untangle to his team. as he attempted cried Kaasen He offtook every of and searched inch his mittens until he found it. bare-handed snow With this news,With of a group from Trail along the Iditarod a relay organized mushers the serum was distributed) Nome. where to (the town Nenana went musher Each miles along the trail twenty with one hundred and between the serum strapped tightly his sled.to and their dogs a terrible through The mushers fought winter their way storm. temperatures, They battled extreme deep snow, wild from animals. and threats against left in this race time. was the last musher Kaasen Gunnar When Nome’s doctor diagnosed doctor the first cases of Nome’s When diphtheria, and deadly a contagious and lungs,disease that affects the throat he panicked. his treat the serum to Without patients, die. surely they would epidemic proportions. reach The disease could used He a telegraph call for help. to was not good. The answer and subzero The snow Nome. fly the serum to to made it impossible temperatures 1925, In most planes had open cockpits, flying made which and dangerous. difficult in bad weather It was January ofIt 1925. A terrible that month in the had occurred diphtheria outbreak ofoutpost town Nome, Alaska. small village This of had been a bustling town twenty of at the end duringthousand people the century. the nineteenth Now, souls remained. hardy than fourteenfewer hundred the closest get to to only way The city, Anchorage, Trail. on the Iditarod travel was to to trip Nome A one-way from about one (called a musher) dogsled the most experienced driver even took Anchorage complete. month to The wind him. whipped around thirty to had plummeted degrees The temperature zero,below his dogs he pushed but relentlessly. Nome. reach he must knew Kaasen It ofwas a matter life or death. “Come on,“Come Balto. can do it!” You the as he fought keep to Kaasen Gunnar screamed Trail. dogsled on the Iditarod Stanford 9 Practice Test Copyright © by The McGraw-Hill Companies, Inc. 15 14 13 D C B A forthispassage? title beanothergood thesemight Which of J H G F by thefinalparagraph? supported conclusions is thefollowing Which of D C B A How didthedoctor firstrequest help? “The Bravest“The Drivers” Dogsled “Learning How to Drive aDogsled” andSurvival” Sport “Dogsled Driving: Strength” “Gunnar Kaasen’s of Story seventy-five mushers modern more than The race usuallyattracts difficult. The Iditarod Trail race isconsidered unchallenged. The 1925speedrecord remains the Iditarod trail. Medicines along are transported still driverdogsled In averbal messagedelivered by By airplane telegraph By dogsled messagedeliveredIn awritten by 33 17 16 18 J H G F thispassage? of third paragraph What doestheword D C B A thepassage? of first paragraph Which didtheauthorusein device J H G F helpful to someonereading thisstory? would thefollowing be Which of Strong Transparent Sickly Weak Metaphor Figurative Language Suspense Flashback on theIditarod Trail aboutarecent story race A newspaper spreads Information abouthow diphtheria A mapdetailingtheIditarod Trail Information aboutsleddogs hardy mean inthe GO ON GO most Copyright © by The McGraw-Hill Companies, Inc. GO ON

34

A Surfing LegendA Surfing

When he was a small he was a child,When out him father took Kahanamoku’s Duke and, the ocean onto custom, following an ancient Hawaiian him tossed the waves.into Duke, For sink or swim. it was swam. He that day From on, left the water. he rarely Fearless, of respectful always yet the sea, he on his surfboard. harnesslearned its power to down flew the steep, He down a mountain.watery a skier careening slopes like as the Known of“father surfing,” as the the world surfers around by is revered Duke best surfer ever. 1890,August Born in to and States the United surfing brought to Duke 1900s. in the early the world Waikiki, at up on the beach grew He swimming and surfing tirelessly. Obsessed with riding wave the next skillfully, and more faster his own surfboards. constructed he even Other most his boards—his imitate in vain to attempted surfersthe beach on that weighed one surfboard sixteen-foot famous was a tremendous and fourteen pounds. hundred Oddly, him the opportunity surfing enlighten the that gave to it wasn’t about the sportworld of surfing—it was swimming. on the was Duke U.S. swim in the 1912 Olympics team in Stockholm, Sweden, he where instant and achieved freestyle event for the 100-meter the record broke the gold medal. fame when he won 1920,In at the event another gold medal in the same won Duke Antwerp, Belgium Olympic games. Olympiads, the two Between Duke the globe, around surfed his way following of gaining a huge devoted of and love fans and sharing his knowledge surfing with thousands. He performed unheard-of surfboard, on his feats considered waves in often attempt.deadly to too build surfboards to taught surfers how novice He and popularized the laid-back surfer attitude and lifestyle. fame His of with exponentially display each increased and athletic prowess aquatic talent. signed him for movie roles, executives Hollywood royalty upon him, attention lavished and celebrities sought him out. Yet, than the land. more the water loved ultimately Duke day, after Day tightly with his hugged the water to his board sprint toward he would swells. the huge out into the breakers through his way and force chest intently,Paddling wait patiently for the right he would wave, and when than most people it—with graceit came he took and confidence more imagine.could 1966, In his death, before years two was inducted Duke of Hall the Surfing into Fame. of Halls is also in the Swimming He Fame Lauderdale, and Fort in Honolulu and the U.S. Olympic of Hall Fame. importantly,Most on in the hearts of his legend lives surfers everywhere. Stanford 9 Practice Test Copyright © by The McGraw-Hill Companies, Inc. 21 20 19 D C B A Duke’s life? characterizes thefollowing Which of J H G F first? happened thefollowing Which of D C B A thatthereader understands— granted thispassagetakes itfor The authorof existence. He lived aconservative andagitated national sport. He introduced a andpopularized celebrity. He andwon his pursued hisfortune theOlympics. He wasaproponent of competition Duke won aFort Lauderdale surfing Duke swaminStockholm Duke roles wascastinmovie Fame Hall of Duke wasinducted to theSurfing how surfersselectabeach how Duke reached Europe asurfboard the construction of how waves beyond form breakers 35 23 22 24 Based on the passage, what isa what Based onthepassage, D C B A reasonable isa which Based onthestory, J H G F thatthereader understands— granted thispassagetakes itfor The authorof J H G F surfers. Duke by known modern isstillwidely Olympics. Duke won aSilver Medal inthe1912 Duke wasalsoanexperienced skier. surfboard. of brand Duke preferred usinganimported team swim how Duke gotonto theU.S. werewhere the1912Olympics held how Hawaii becamethefiftiethstate what surfingis A type of swimming stroke swimming of A type act A daring A body part A noise conclusion? GO ON GO feat ? Copyright © by The McGraw-Hill Companies, Inc. GO ON is imminent! All orders are due by are due by All orders s Day parade s Day ’ Saint Patrick s Pizza Kits be accepted after that date. No exceptions. be accepted after that date. ’ 36 15 minutes. All items are so delicious that they will ± One entry into $300 cash raffle WIN! Fabulous Fudge Fabulous One entry baseball cap and a school into raffle and a portable CD player. Items for Sale Items for Bombshell Brownies Five entries into raffle, a school baseball cap, a gym bag, a school baseball cap, entries into raffle, Five (12pc/box) $6.95 per box $6.95 (12pc/box) to participate in the (16pc/box) $7.25 per box $7.25 (16pc/box) Two entries into raffle, a school baseball cap, and a gym bag a school baseball cap, entries into raffle, Two WIN! Mama Scarano WIN! You Have to Sell to Win! Have to Sell to You ABSOLUTELY WILL WILL NOT ABSOLUTELY A Summons to All Musicians A Summons WIN! Sell 1—3 items See you in New York! Orders New York York New City Inform your customers that items will arrive on February 25. customers your Inform Sell 4—8 items Our mission is to raise $3,000 for travel, food, and hotel expenses. hotel expenses. and food, travel, $3,000 for raise Our mission is to Contact Barbara Samms, our parent coordinator, at (987) 654-3210 for more details. more details. at (987) 654-3210 for our parent coordinator, Samms, Contact Barbara (contains 3 crusts, and 3 sauce and cheese packets) $15.95 each and cheese packets) (contains 3 crusts, and 3 sauce Sell 9—12 items All items are delivered frozen and nonperishable. They may be kept in the freezer for up to three : If every band member sells twelve of the items below, we can easily meet this goal. this goal. can easily meet we of the items below, band member sells twelve If every

Sell more than 12 items Additional information and color brochures for each product are included in the enclosed materials. each product are included in the enclosed materials. and color brochures for Additional information

months. Baking time for pizza is approximately 12 sell themselves. NOTE Money should be collected when the order is taken. Customers should make checks payable to The Big to payable checks Customers should make when the order is taken. should be collected Money be placed All orders must to tally all orders. are included in each packet Order forms Apple Band Fund. Please provide should be returned and only these forms on these forms the end of the selling period. at provided. totals on the forms each item sold and include these totals for grand Tuesday, January 5. 5. January Tuesday, Our annual tripOur annual to Stanford 9 Practice Test 25 It is implied by the flyer that each band 28 Which of these does the flyer resemble member — most? A knows about the trip to New York F A recipe B will sell more than twelve items G An outline C has already sold a few items H A set of instructions D is a very talented musician J A calendar

26 Which of these is an opinion from the 29 If you sell five items, you will win — poster? A a baseball cap F All items are delivered frozen. B a gym bag G All orders are due by January 5. C a portable CD player H Items may be kept in the freezer. D a raffle ticket only J All items are so delicious that they will sell themselves.

30 Which of these will not be covered by the $3,000 from the sale? 27 You can tell from the flyer that deadlines F Travel are — G Food A insignificant H Hotel expenses B flexible J New band uniforms C important D nonexistent Copyright © by The McGraw-Hill Companies, Inc.

37 STOP Copyright © by The McGraw-Hill Companies, Inc. e, . Tom. est o W ns nic io ding t and GO ON r o uest c ast, E ac th, ng, ou o lo S o th, r o N Language compass: North, South, East, and West. compass: north, south, east, and west. compass, north, south, east, and west. as is Correct questions nicely, so nice,question’s so so questions nice as is Correct too long,too too according long,to to according long,to too according as is Correct we left him a big tip. we mpass, o A B C D our q answered The waiter so There are four directions marked on the directions four marked are There c F G H J The story was t A B C D 5 3 4 38 ho wants carved on a house in als r ht ume oug e b v man n . o r ha ? st o fir “Who wants to go first?” go wants to “Who go first wants to Who go first?” wants to “Whom as is Correct numerals roman numerals Roman Numerals Roman as is Correct Tim and her has bought Tim bought and she have Tim and her has bought Tim as is Correct Correct as is Correct im and he o g There were r T A F G H J the cornerstone of the building. B C D t F Florida. G H J The teacher asked the class, asked The teacher w

SAMPLE DIRECTIONS sentence. each Read might There be a mistake portionin the underlined of the sentence. The might punctuation, be in mistake capitalization, usage,word of or a combination those. If you a mistake,discover that best the answer choose the mistake.corrects If find no mistake, you choose 2 1 Stanford 9 Practice Test 6 Even though Sarah was tired, she goes 10 The fog was so thick that we couldn’t see out to play. nothing at all. F gone out F we couldnt see nothing G went out G we could not see nothing H going out H we couldn’t see anything J Correct as is J Correct as is

7 My two most difficult subjects math and 11 The dinner bell rang it was time to eat. science were canceled today. A rang; it was A subjects, math and science, were B rang, it was B subjects math and science, were C rang, it is C subjects, math and science were D Correct as is D Correct as is

12 The best stories usually involve 8 I got to finish this job soon so I can go interesting characters in its plots. to the party. F characters in it’s plots F I got to get finished G characters in their plots G I must finish H characters in the plots H I have to be finishing J Correct as is J Correct as is

9 The apples in Janets’ backyard always taste good. Copyright © by The McGraw-Hill Companies, Inc. A apples in Janets’ backyards B apples in Janets’s backyard C apples in Janet’s backyard D Correct as is

39 GO ON Copyright © by The McGraw-Hill Companies, Inc. GO ON if is correct. the sentence The sun was shining on the lake,The sun was shining the go swimming. to girls prepared The sun, was shining on the lake. go swimming. to The girls prepared The sun was shining on the lake. go swimming. to The girls prepared as is Correct Offering is a trumpet lessons now Harmon. James musician professional Harmon,James a professional musician, offering trumpet is now lessons. Harmon,James a professional now is offeringmusician trumpet lessons. as is Correct James Harmon,James a professional offering is now trumpetmusician lessons. The sun was shining on the lake The sun was shining the girls go swimming. to prepared Correct as is Correct A B C D F G H J SAMPLE DIRECTIONS in for an error in the box the sentence Study structure.sentence choice answer the Mark rewrite the sentence, to showing the best way or mark 16 40 that I’m m as and e l the h p to give us l e n t kso spinac ac ht, r J e nig o ffic . oli c l ask o

asks tell them asks tell ask, them tell them ask tell as is Correct We’ll ask Officer Jackson Jackson ask Officer We’ll jackson ask officer We’ll jackson ask Officer We’ll as is Correct tonight, spinach, peas, and broccoli tonight: peas and broccoli spinach tonight: spinach, peas, and broccoli as is Correct c ’l o

e r There are three different vegetables on vegetables different three are There t the menu A not here. B C D b If asks, Todd and Hiro A directions to the festival.directions to B C D W G H J F

Stanford 9 Practice Test 15

14 13 Copyright © by The McGraw-Hill Companies, Inc. 19 18 17 D C B A J H G F D C B A at once inonesitting. I read itall wassogood, The book running errands. running and take weekend trips, work, thecarto driveShe bought to to goto themovie. soshewasallowed homework, her Alice wasfinishedwith Correct asis and finishherhomework. Alice wasallowed to goto the movie movie. because shewasallowed to goto the herhomeworkAlice finishedwith Correct asis errands. andrun take weekend trips, thecarto driveShe bought to work, errands. running and take weekend trips, work, to thecarfordriving She bought errands. andran take weekend trips, thecarto driveShe bought to work, Correct asis one sittingallatonce. I read itin The bookbeingsogood, one sitting. I read itin The bookwassogood, all atonce inonesitting. I read it The bookIread was sogood, being allowed to goto themovie. herhomework,Alice wasfinishedwith 41 21 20 22 J H G F D C B A J H G F I candoittomorrow. Sanchez thelibrarian. ’ o uyt oi oa,Ithink I’m too busyto doittoday, I took my reading listto Mr. bakery. store andbread atthe grocery lemonsatthe Sheila bought Correct asis Sanchez. Mr. I took my reading listto theLibrarian who isthelibrarian. Sanchez I took my reading listto Mr. the librarian. Sanchez, I took my reading listto Mr. Correct asis do ittomorrow. Ithinkcan Not ableto doittoday, think Icandoittomorrow. I’m too busyto doittoday andI can doittomorrow. Ithink I’m too busyto doittoday; Correct asis store atthegrocery andbakery.bought Lemons andbread were whatSheila lemonsandbread atthebakery.bought storeThe grocery iswhere Sheila store forbread. andthebakery lemonsatthegrocery Sheila bought GO ON GO Copyright © by The McGraw-Hill Companies, Inc. GO ON . dinosaurs became All way to make the last two sentences last two the make to way 42 Tyrannosaurus rex Tyrannosaurus best and the meat-eating and the meat-eating All dinosaurs became extinct about sixty-six about ago,All dinosaurs became extinct million years as did other land and sea animals. many sixty-six about ago,All dinosaurs became extinct million years many other land and sea animals did, too. sixty-six about ago,All dinosaurs became extinct million years so did other land and sea animals. many sixty-six about ago and All dinosaurs became extinct million years at that time as well. animals became extinct other land and sea many Which ofWhich the following is the into one sentence? into A B C D

Brachiosaurus

extinct about sixty-six ago.extinct million years sea animals did, other land and Many too. Dinosaurs were large reptiles that ruled the Earth than one hundred large reptiles for more Dinosaurs were million years. largest land animals, the them were Among as the plant-eat- such ing SAMPLE

DIRECTIONS paragraph each that follow. and the questions Read question. for each answer the best Mark Stanford 9 Practice Test Copyright © by The McGraw-Hill Companies, Inc. 24 23 crowd inthestandsto cheer. Thiscausedthe thecheerleaders jumpedandshouted. Onthesideline, line. astraight and formed forms contrasted sharply b contrastedforms sharply sunshinethered uni- In the bright The bandbeganto play asthefootballteam took thefield. J H G F combine thelasttwo sentences? Which of the following isthe thefollowing Which of D C B A sentenceopening fortheparagraph? would thefollowing bethe Which of the stands. and shouted thecrowd cheering in On thesidelinecheerleaders jumped the standsto cheer. and shouted andcausingthecrowd in cheerleaders jumped On thesideline, stands to cheer. causingthecrowd inthe and shouted, cheerleaders jumped On thesideline, the crowd inthestands. acheer fromand shouted bringing cheerleaders jumped On thesideline, and fast. Football players need to bestrong championship game. It wasaperfectday forthe the players ontheteam. Coach Avery popularwith wasvery protect theirheads. Football players wear helmetsto y h epgeno h rs.Tepaesrnt h etro thefield Theplayers to ran thecenter of thegrass. the deepgreen of best way to Paragraph 1 Paragraph best 43 25 D C B A wordthe underlined intheparagraph? Which word isthe if at with on best replacement for GO ON GO Paragraph 2 Without elevators, there would be no New York City skyline. More than any other technological advance, it was the invention of the elevator that made skyscrapers possible. People have been able to build huge structures for a long time, but without an easy way to reach the top, such structures could not be used for very much. Could you imagine living on the thirty-first floor of a building if you had to walk up the stairs every day to get home?

26 Which of the following is the most 28 Which of the following best describes interesting way to write the underlined the main point of this paragraph? phrase in the paragraph above? F To point out the importance of the F since a long time ago elevator G for many years G To describe the invention of the H since the time of the pyramids elevator J for some time before now H To discourage people from living in tall buildings J To explain why tall buildings are more useful than shorter buildings

27 Which of the following sentences does not fit in the paragraph? A The first skyscrapers were built about

one hundred years ago. Copyright © by The McGraw-Hill Companies, Inc. B The Empire State Building is eighty- six stories tall. C Buildings that are hundreds of feet tall are now common. D The invention of the automobile made long-distance travel easier.

Stanford 9 Practice Test 44 GO ON Paragraph 3 Tall grasses waved gently in the soft, warm breeze. Louisa walked cautiously to the edge of the high cliff. Sitting on a large rock, she set up her easel and canvas and took her pencils, brushes, and paints out of her backpack. Far below, silver waves crashed upon a black and rocky shore. Louisa took a drink of water from her canteen and prepared to begin her painting.

29 Which is the best closing sentence for 30 Which of these would not fit in this this paragraph? paragraph? A Louisa had always loved painting in F Louisa enjoyed listening to the waves the open air. crash against the shore. B There was not a single cloud in the sky. G Like her favorite painter, Gustave C Far out on the horizon, a boat Courbet, Louisa enjoyed working in chugged along. nature. D A cricket hopped upon Louisa’s shoe H High cliffs are important in several of and chirped loudly. Shakespeare’s plays. J The view was truly astonishing. Copyright © by The McGraw-Hill Companies, Inc.

45 STOP 46 Ten Days to the Stanford 9

ay Saturd Friday ursday MARCHay Th Wednesd Tuesday Monday Sunday 7 6 5 4 3 2 1 14 13 12 11 10 9 8 AT-9 S 21 20 19 18 17 16 15 28 27 26 25 24 23 Copyright © by The McGraw-Hill Companies, Inc. 22

31 30 29

47 Copyright © by The McGraw-Hill Companies, Inc. Copyright © by The McGraw-Hill Companies, Inc. • • • • Vocabulary How toPrepareforReading Reading Vocabulary you beready will to tackletheSAT-9! in class, thisstudy guideandpay attention you take advantage of If theSAT-9. of findineachyou section will questions of walkyou through thetypes pageswill following The testsEach different section skills. Ten DaystotheSAT-9 to helpyoudesigned doyour bestontheSAT-9. andtips test-taking strategies you of beintroduced to will avariety Over thenext several days, test. theStanford 9 Welcome to thefinalstageinyour of preparation fortheLanguage portion Arts Ten DaystotheStanford9 Save your indexcards a sentence usingeach word under itsdefinitionto helpyou remember it. anindexcard. each word you don’t know onthebackof Write of definition theprimary up whenyou gethome. andlookit itdown write don’t know, you. indexcards with to carry Always try words you don’t know. on. youmagazine cangetyour hands and newspaper, book, Read every oku h eiiin fallthe Look upthedefinitionsof When you seeorhear aword you • • • • • • Language SA Study Skills Language Spelling Reading Comprehension Reading Vocabulary and test yourself with them. with and test yourself covers thefollowingsixtestsectionsindetail: 3. 2. 1. Reading Vocabulary hasthreeparts: 49 B words inthesentence. determine aword’s meaningby lookingatother Words inContext. the sameword. to distinguishbetween different of meanings Multiple Definitions. word thatmeansthesameassampleword. Synonyms. A C These questionsaskyou to picka These questionsaskyou to These questionsaskyou Write Reading Vocabulary Part 1: Synonyms Different Words that Mean the Same Thing Each question in the Synonyms part of the SAT-9 gives you a phrase with an underlined word and four answer choices. Your goal is to decide which of the answer choices has the same meaning, or close to the same meaning, as the underlined word.

Each Synonyms question will look like this: Rueful means — “A wise man hears one word ? A kind-hearted and understands two.” B rude C sorrowful —ancient proverb D excited Here is how to approach this question: 1. Cover the answer choices so you are not confused by words that are similar to the underlined word. For example, even though the words rueful and rude both begin with “ru-,”they do not mean the same thing. Some wrong answers are included just to confuse you. These answer choices are called distractors.

2. Try to come up with your own definition or synonym for the underlined word. For the above question, you might think: A rueful person is someone who is sad.

3. Uncover the answer choices and get rid of the choices that you know are wrong.

4. Look at answer (A). Does the word kind-hearted mean about the same as sad? No, it doesn’t. Eliminate answer (A).

5. Look at answer choice (B). In step one, we determined that (B) is probably there to Copyright © by The McGraw-Hill Companies, Inc. distract you, but we should consider it anyway. Does rude meant the same thing as rueful? Rude means “disrespectful.” Eliminate answer (B).

6. Now look at answer (C). Does sorrowful mean the same as sad? Yes, it does. Keep answer (C).

7. Go on to answer (D). Does excited mean the same as sad? No, it doesn’t. Eliminate answer (D). Now you can be sure that answer (C) is correct!

Now try a harder one:

To debase something is to — ? A mistake it B corrupt it C endure it D mention it

Ten Days to the Stanford 9 50 Copyright © by The McGraw-Hill Companies, Inc. Some wordscanbeusedinmorethanoneway. Forexample: Different MeaningsoftheSameWord Reading Vocabulary Part2:MultipleDefinitions this: Can youcomeupwithyourowndefinition?Ifnot,try The word The word The word 3. 2. 1. 7. 6. 5. 4. milk milk milk osnt lmnt hsase lo Now you canbesure thatanswer (B)iscorrect! Eliminate thisanswer also. does not. it have anything deception? incommon No, with Does “mention it” Look atanswer (D). answer (C). of rid Get doesn’t. it have anything deception? incommon No, with Does “endure it” Look atanswer (C). unless you are aboutaword’s certain meaning. theanswer choices always read therest of However, This isprobably thecorrect answer. deception have anything theword incommon Doesthephrase with “corrupt it” Look atanswer (B). deception.common with hasnothingin Thephrase “mistake it” answer (A). you out canrule First, that are wrong. obviously Eliminate theanswers Uncover theanswer choices. deception You remember might itfrom theword before. Think aboutwhere you have seentheprefix “de-” undo.” which isaprefix thatmeans “to theword, of part “de-” you recognize might the In thiscase, you recognize. theunderlinedword hasany that parts if Ask yourself Cover theanswer choices. choose your answer. in the sentence above is used asan aboveisused in thesentence asa aboveisused in thesentence in the sentence above is used asa aboveisused in thesentence I canprobably milk Grandma always makes milk medrink I wantto buy amilk e,i os hnsmtigi orp,there isusuallydeception involved. When somethingiscorrupt, itdoes. ? Yes, Keep theword . deception my work. bossfor extra machine. verb noun adjective 51 in mindasyou meaning“ask”or “demand.” , meaning“adrink.” , meaning“milk-producing.” , . “‘When aword,’ Iuse —Lewis Carroll less.”nor mean—neither more to it choose I what just means ‘it tone, in arather scornful said Dumpty Humpty Each Multiple Definitions question will look like this: The author was acclaimed for her Helpful Hint ? ability to thread several subplots into her novels. If the sample word is a noun, the correct In which of the following does thread mean answer will be a the same thing as in the sentence above? noun. If the sample A The tailor sat patiently trying to thread word is a verb, the the needle. correct answer will B The painter tried too hard to thread be a verb. If the reality and fantasy into his paintings. sample word is an C The new sheets are made with very soft adjective, the correct thread. answer will be an D It is too difficult to follow the thread of adjective. such a long play.

Here is how to approach this question: 1. Look at the underlined word in the sample sentence and decide if it is used as a noun, verb, or adjective. Ask yourself these questions: • Is the word an object? If so, it is a noun. • Is the word an action? If so, it is a verb. • Does the word describe? If so, it is an adjective. In the above example, the sample sentence uses the word thread as a verb. Make a mental note of this—it will help you find the correct answer.

2. Read the sample sentence again. Imagine someone performing the action that the sample

sentence describes. Copyright © by The McGraw-Hill Companies, Inc.

3. Look at each answer choice. Decide if each underlined word is used in the same way as the word in the sample sentence. Since thread is used as a verb in the sample sentence, the correct answer choice will also use it as a verb.

4. Start with answer (A). In this sentence thread is also used as a verb, but be careful! This verb refers literally to the act of threading a needle; the sample sentence uses the word thread figuratively to describe an author’s method of writing. Watch out for distractors that are the same part of speech but have different meanings. Eliminate answer (A).

5. In answer (B), thread is used as a verb that describes an artist’s method of painting. Does answer (B) use thread in the same way as the sample sentence? Yes, it does. Keep answer (B).

6. Even though you think you’ve found the correct answer, you should always read the remaining answer choices just to be sure. Look at answer (C). In this sentence, thread is used as a noun that refers to a type of sewing material. Eliminate answer (C).

Ten Days to the Stanford 9 52 Copyright © by The McGraw-Hill Companies, Inc. be aclue. What does Here iswhatarealWords inContextquestionwilllooklike: For example,youwillneverseethistypeofsentence onthetest: Look fortheclues! theword. always There will becluesinthesentence to helpyou figure outthemeaningof scare you. butdon’t letthat thesewords seemunfamiliarto you, will Many of looking atthewords around it. anunderlinedword inasentence by Words inContext questionsaskyou to determine themeaningof Reading Vocabulary Part3:Words inContext Multiple DefinitionsTips The subject matter in both sentences is about the same. However, the first sentence uses theword uses sentence thefirst However, isaboutthesame. matterinbothsentences The subject Watch tricks! outforthese theword sentence, the first The word them! Keep peeledfor youreyes hasclueslike that. inthissection sentence Every that theroom really stinks. Now canyoutellwhat noun , while the second sentence uses theword uses asa sentence thesecond while , 8. 7. cast fetid Here’s anexample: speech. but which usestheunderlinedword of asadifferent part to thesamplesentence, Watch outfordistractors! o’ nwtease,gesadmv n o’ e orefgetstuckonone question! Don’t letyourself guessandmove on. don’t know theanswer, you still If answer. pickingtheright answer choices helpimprove will your chances of Eliminate asmany answer choices asyou can. meanings. have andstill speech different two of words bethesamepart might Remember, Now you canbe sure thatchoice (B)iscorrect! Eliminate answer (D). aplay. development of In thissentence, Now lookatanswer (D). My brother’s room issof My brother’s room isf is used asa is used mean? There is no way to tell from looking at this sentence. On the test, there will always there will the test, On mean? There isnoway totellfrom lookingatthissentence. fetid hncosn orase hie,watch outforwords like this: When choosing your answer choices, I asked to q Arnold r Jonas gave usaq Mr. ’ ebro thecast I’m of amember Corey’s legisinacast noun cast en?Sne“ aet odm oeweee ’ ntee”youcantell means? Since I’m“I have inthere,” toholdmy whenever nose en patrml. ntescn etne tmas“ru factors.” it means “group of In sentence, thesecond means “plaster mold.” ntesnecsaoe u thsadfeetmaigi ahsnec.In but ithasadifferent meaningineachsentence. above, in thesentences e t id e t . Sometimes there will beananswerSometimes there choice will similar thatisvery id that Ihave to holdmy I’m nosewhenever inthere. uiz uiz . me. . 53 verb . Don’t tricks! befooledbythese . thread vngtigrdo oneortwo wrong Even of gettingrid is usedasa noun that refers to theplot quiz as a Each Words in Context question will look like this:

Mr. Purdue thought the young artist’s work was base and needed to be more “All my life I’ve looked ? sophisticated and refined. What does base mean? at words as though I A Elevated were seeing them for B Expensive the first time.” C Colorful —Ernest Hemingway D Crude

Here is how to approach this question:

1. Cover the answer choices so they don’t confuse you. All of the answer choices seem to fit, but only one of them is correct.

2. Search the sentence for clues to the meaning of the underlined word. From the above question, you know that the word base refers to artwork that could be better if it were more sophisticated and refined. These are important clues!

3. Based on the clues, try to think of your own word or phrase for the underlined word. In this case, you might think: Artwork that is not sophisticated and refined is primitive.

4. Look at answer (A). Do you think the artist’s work was elevated? No, elevated means “higher up.” Eliminate answer (A).

5. Go on to answer (B). Do you think the artist’s work was expensive? No, the word base refers to the artwork, not to its price. Get rid of answer (B).

6. Look at answer (C). Do you think the artist’s work was colorful? It might have been, but

the color of a work of art has nothing to do with its sophistication or refinement. Get rid Copyright © by The McGraw-Hill Companies, Inc. of answer (C).

7. What about answer (D)? Do you see any clues in the sentence that tell you the artist’s work was crude? Yes! If artwork needs to be more sophisticated and refined, it is probably because the work is crude or primitive. The correct answer is (D)!

Helpful Hint Some words automatically make us think of other words. These words are called association words. Association words are another kind of distractor. If you had not been careful in the example above, you might have associated the words expensive and colorful with artwork. Watch out for association words!

Ten Days to the Stanford 9 54 Copyright © by The McGraw-Hill Companies, Inc. Tips toRememberforReadingVocabulary Be suspicious. Take your time. carefully. Don’t finishearly. themwrong! thequestionsandgettingmany of is always better of thanfinishingall Read carefully. question each you have time. back to thedifficultquestionslater if Go move onto thenext question. you don’t ananswer come fairlyquickly, upwith If Don’t by difficultquestions. become frustrated meaning. theword thatyou recognize. of to findparts try aword, you don’t know themeaningof If you read on. better answer if theanswer choices before choosing your answer. Read allof words thathavehelp you different avoid meanings. by beingtricked “look-alike” your before you definitions own lookattheanswer choices. Try to thinkof oeie rfx otwr,or suffixcantell you alotaboutword’s root word, Sometimes aprefix, okfrcusadb ayo distractors. of Look forcluesandbewary Many thetest even timeto allows studentsrush though read enough opeigms fthequestionsandgettingthemright Completing mostof 55 An easier question will becomingAn soon. easierquestionwill Sometimes you finda will This will Reading Comprehension

The Reading Comprehension portion of the Reading Comprehension has three SAT-9 asks you to read a variety of passages types of passages: and then answer questions based on what 1. Narrative Passages. These are either you read. The passages appear roughly in fictional stories or stories based on an order of reading difficulty. This means that historical event. each passage you read will be a little harder than the one before it. 2. Informational Passages. These types of passages are similar to what you read in your textbooks. They will tell you about How to Prepare for Reading people, places, or things. Comprehension 3. Functional Passages. These are the kinds of • Read, read, read. As with the Reading text that you see in real life. They are usually Vocabulary portion of the test, the best in the forms of advertisements, schedules, way to prepare is by reading as much as or flyers. you can. • Put more effort into understanding what you read. Stop to summarize while you are reading. Ask yourself: What is the main idea of this passage? • When you see a word you don’t know, try to learn its meaning from its context. After you’ve guessed at the word’s meaning, look it up to see if your definition is correct. Write your new word on the front of an index card and write its definition on the back. • Study your vocabulary index cards. See if you can look at the words on the front of your index cards and tell yourself the correct definition. Then really challenge yourself and do the reverse! The more words you know, the more likely you are to understand what you read. Copyright © by The McGraw-Hill Companies, Inc.

Helpful Reading Comprehension Terms Main idea—What a passage or paragraph is mostly about. Theme—Another word for main idea. Quite often, there is a lesson that can be learned from the theme of a passage. There may be more than one theme in any given story. Summary—A brief statement of the main points covered in a passage. Setting—The time period and place in which a passage happens. Figurative language—A colorful and creative way of writing something that makes it more interesting to read.

Ten Days to the Stanford 9 56 Copyright © by The McGraw-Hill Companies, Inc. Here iswhatyoushoulddowhenreadinganInformational Passage: atextbook. much like thecontent of contain They many andare facts very orthings. places, Informational Passages discussreal-life people, Informational Passages Here iswhattodowhenreadingaNarrativePassage: events bebasedonhistorical orpeople. will sometimes they but Narrative Passages always will soundlike stories, fictional story. A Narrative of Passage isatype Narrative Passages 2. 1. 2. 1. 7. 6. 5. 4. 3. reader? to getacross What to the wastheauthortrying Think aboutthepassage’s themeorlesson. sodon’t waste your timereading it. be questionedonthis, You not will thepageaboutauthorandillustrator. atthebottomIgnore of theprint when you needthem. helpyou It will to locate answers each paragraph. themainideaof Make amental note of • • • Ask yourself: Think aboutwhoistelling thestory. • • • • • • • thesequestionsasyou read: Ask yourself details! important missvery you will information, When you justscanapassage forbitsandpieces of Don’t justskimthepassageforanswers. Read theentire passageslowly andcarefully. the story. of thatsometimesthetitleisnotbestsummary however, Remember, astory. idea of helpyou Sometimesthetitlewill focusonthemain Read thetitlebefore reading thestory. when readingwhen anarrative passageto make sure thatyou you understandwhat have read. thesamequestionsyou would Ask yourself Read theentire passageslowly andcarefully. thepassage. tell you Thiswill thetopic of Read thetitle. What point is the author trying to makeWhat by having thispersontell thestory? pointistheauthortrying how? so, Is related thenarrator to thecharacters? If Who isthenarrator? How end? doesthestory Who are thecharacters? Where take place? doesthestory When domajorevents happen? What happenslast? What happensfirst? What happensinthestory? 57 “Be sure“Be that you go to —John Ruskin yours.” to find not meaning, his at get to author the 3. Make a metal note of the main idea of each paragraph. Read the topic sentence of a paragraph again if you get stuck.

4. Don’t try to memorize every single detail. You’re not a computer! Just try to remember where blocks of facts are located in the passage so you can go back and find them when you need them.

5. As with the narrative passage, ignore the print about the author and illustrator.

Functional Passages Functional Passages look like writing you see in everyday life, like flyers, advertisements, letters, schedules, and posters. Functional Passages often provide information about things, places, or events.

Here is what you should do when reading a Functional Passage:

1. Read the title.

2. Read the entire passage carefully—don’t just scan it, but don’t try to memorize all the details.

3. Ask yourself these questions as you read: • What is the purpose of this passage? “Look twice before • Does the passage inform you about something? If so, you leap.” what? • Does the passage try to persuade you to do —Charlotte Brönte something? If so, what? 4. Don’t ignore the small print in functional passages. It is often very important, and you might be questioned on it! Copyright © by The McGraw-Hill Companies, Inc.

Reading Comprehension has four basic types of questions: Each reading passage is followed by a series of questions. The questions are not in order of appearance in the passage. They are also not in order of difficulty. 1. Stated Information Questions. These questions ask you to recall details that were mentioned in the passage.

2. Drawing Conclusions Questions. These questions ask you to form ideas based on hints from the passage.

3. Passage Analysis Questions. These questions ask you to make judgments based on information from the passage.

4. Reader Approach Questions. These questions ask you to recognize specific qualities of the text itself.

Ten Days to the Stanford 9 58 Copyright © by The McGraw-Hill Companies, Inc. • • • • • • Here aresomethingstoconsiderwhenfacedwithStatedInformationquestions: Here aresomeStatedInformationquestionsfromthePracticeTest: well you recall details. The Stated Information questionsrefer to detailsstated inthepassageandare to test designed how Stated InformationQuestions ? ? at.Don’t rely onyour memory. facts. fyuaecnue yaqeto,restate itinyour words own to make itclearer. you are confused by aquestion, If Keep aneye outforquestionsthatusethewords apply. Remember to scantheentire passageforalldetailsthatmight different places inthepassage. from information require youSometimes questionswill to combine two ormore pieces of Don’t letthatconfuse you! The correct answer choice often beworded will alittledifferently thanitappearsinthepassage. choices. Thenlookattheanswer inyour words. own repeat itto yourself Once you’ve foundtheanswer, you thinkyou know theanswer, Even if obviously wrong first. wrong obviously read carefully! Remember toeliminatetheanswersthatare so them canbeconfusing, questions These Helpful Hint All of the following answers are true answers are true thefollowing All of Which of the following is thefollowing Which of getting fooledbydistractors! look backatthepassagetocheckinformationandavoid passage thatistakenoutofcontextjusttotrickyou.Always wrong answerchoiceswillcontainsomefactordetailfromthe answer choicesaredistractors.Thismeansthatmostofthe In theReadingComprehensionsection,almostallofwrong Where didthe How didthedoctor firstrequest help? litters? who visitor happentoWhat apark will first musher get the serum in1925? musher gettheserum not mentioned inthe passage? mentioned always except 59 refer backto thepassageto double-check the except which one? which and not For example: . Drawing Conclusions Questions The Drawing Conclusions questions ask you about things that were hinted at in the passage. To choose the correct answers, you will have to look for clues in the passage.

Here are some Drawing Conclusions questions from the Practice Test: • Some questions ask you to decide on the “main idea” of the passage or to pick “another good title” for the passage. Here are some examples:

What is another good title for this passage? ? What is the main idea of the third paragraph?

Here’s how to answer main idea or title questions:

1. Look at the entire passage, including the title.

2. Try to think of a word or phrase that describes what each paragraph is mostly about.

3. If a question asks about the entire passage and an answer choice only covers one or two paragraphs, it does not describe the passage as a whole. A too-specific answer choice like this might be listed as a distractor. Eliminate answer choices that are too specific.

4. Even the correct answers are not always perfect. Sometimes you just have to pick the answer that is the least rotten! • Some questions will ask you to determine the meaning of a word or phrase in the passage. Here are some examples from the Practice Test:

In the last sentence, the word ingrained most nearly means — ? In the second paragraph, what does pursue mean? Copyright © by The McGraw-Hill Companies, Inc.

Here’s how you should answer questions that ask you to determine the meaning of a word or phrase: 1. Find the word or phrase where it appears in the passage.

2. Reread a few lines above and a few lines below the word or phrase to understand its context.

3. Look for clues in the surrounding text that will help you decide what the word means. Remember—there will always be a clue!

4. Eliminate the answers that you know are wrong, and pick the best answer from what is left.

Ten Days to the Stanford 9 60 Copyright © by The McGraw-Hill Companies, Inc. • • Some differenttypesofPassageAnalysisquestionsaredescribedbelow: answer thesequestions. statement general to have you will you to what turn have read of into somesort For themostpart, These questionsrequire you to provided make basedontheinformation inthepassage. judgments Passage AnalysisQuestions Here aresomethingstoconsiderwhenfacedwithDrawingConclusionsquestions: ? ? Passage Analysis questions, eliminating wrong answers is very important! important! answersisvery wrong eliminating Passage Analysis questions, With answersfirst. Eliminatethewrong followasanswerchoices. will general statements Several For example: passage. beasked you to will draw conclusion ageneral you basedonwhat have read inthe Most often, pnos For example: opinions. You beasked questionsthattest will your to tell ability thedifference between and facts ic fifrainta stu eadeso anyone’s orfeelings. thoughts regardless of thatistrue information a pieceof thatan remember For questions, these Helpful Hint that arenotincluded inthestory. story. Don’t be fooledbyfactsandopinionsthatyou recognize,but In factandopinion questions, thecorrectanswerwillalways beinthe o oeo thesequestions, For someof ttd but itisalways from by evidence thepassage. supported stated, Theanswer isnotalways you may have to doalittle thinkingonyour own. questions, For these Some questionsaskyou to draw conclusions aboutacharacter’s personality. toinformation thecorrect come answer. upwith you may have to lookforcluesinmore thanoneplace andcombine the these questions, For oraschedule. aflyer, You sometimesbeasked questionsaboutanadvertisement, will really well! Just eliminate theworst answers andchoose from whatisleft. There is enough information inthepassageto show information that — There isenough hc ftheseisan Which of opinion opinion none in thepassage? of the answer choices will seemto theanswer choices answer thequestion will of 61 is based onsomeone’sis based anda orfeelings thoughts fact is • Some questions will ask you about the author’s purpose for writing the story. For example: ? What is the author’s purpose for writing this passage? Think about the passage as a whole and ask yourself: Is it intended to persuade the reader? Does it give a lot of information about a subject? Does it explain a historical event? Look at the answer choices and find the one that is closest to your own.

Reader Approach Questions Reader Approach questions ask you recognize specific qualities regarding the text itself. These questions can be a little tricky because you will not find the answers in the passage. • Some Reader Approach questions ask where you should look to learn more about the subject of the passage. Here’s an example from the Practice Test:

Which of the following would be most ? helpful to someone reading this story? Helpful Hint A Information about sled dogs The moral of a story B A map detailing the Iditarod Trail is the lesson that it C Information about how diphtheria spreads gets across through D A newspaper story about a recent race on the characters and the Iditarod Trail the plot. Most often, the moral of a story Here is how you should approach this question: will not be stated 1. Cover the answer choices so they do not directly. confuse you. Copyright © by The McGraw-Hill Companies, Inc. 2. Decide what would have been most helpful to you while reading the story.

3. Uncover the answer choices and look at (A). Would information about sled dogs have been helpful? It might have been interesting, but since the passage is about a grueling trip over rough Alaskan terrain, answer (A) is not your best choice. Eliminate answer (A).

4. Look at answer (B). Would a map detailing the Iditarod Trail have been helpful? Yes, it would have. As you read the story, you could have followed the dogsled’s progress on the map. This is probably the correct answer.

5. Look at answer (C). Would information about diphtheria have been helpful? Not really. This passage is about the journey caused by the diphtheria outbreak; it’s not about diphtheria itself. Get rid of answer (C).

6. Look at answer (D). Would a newspaper story about a recent race on the Iditarod Trail have been helpful? Once again, it might be interesting, but not especially relevant to this passage. Eliminate answer (D).

7. You can be pretty sure that answer (B) is correct!

Ten Days to the Stanford 9 62 Copyright © by The McGraw-Hill Companies, Inc. Tips toRememberforReadingComprehension Always torefer backtothepassagefindyouranswer! remember orschedule. word cluster, is aflowchart, thegraphic if theanswerchoicesgoesinblankspace—even of which youmustdecide For questions, these Here isanexampleofagraphicorganizerfromthePracticeTest: • • ? based onit. andanswer aquestion information askyou to read ablockof like theexample below, Others, in. andaskyou information to it fill leaveMany organizer questionswill outonepiece of graphic form. passage into avisual from organize someinformation the will Thesegraphics orword clusters. schedules, outlines, Some Reader Approach questionsinvolve organizers like interpreting flow graphic charts, usually sandwiched between sandwiched usually easierquestions. guessandmove on. you don’t know theanswer to aquestion, If Keep thebestanswer choice inmindthatsometimes isjustthe choosing thecorrect answer. Eliminate answer choices wrong. thatare obviously at theanswer choices. inyour words. own itto repeat yourself Once you’ve foundtheinformation, youRefer need. to thepassageforinformation in mistakes! Read andanswer each choice question slowly andcarefully. V TheLast GreatRaceonEarth IV. III. I TheDoctorsendsfor help II. .Kassenonthe Trail I. 63 Do not rely on your memory! Do notrely onyour memory! D C B A What inthemissingentry? belongs h fet ftheGoldRush The effectsof influenza The spread of storm againstthe Kassen struggles outbreakDiphtheria diagnosed This will increase yourThis will chances of Careless result reading will least rotten! Difficult questionsare Then look Spelling Bzzz The goal of the SAT-9 Spelling Test is to assess your spelling skills based on what you have There are six types of spelling learned about the phonetic and structural mistakes on the Spelling test: principles of words. The misspelled words on 1. Homophone mistakes. These questions this test reflect the most common spelling test your ability to distinguish between errors for tenth-grade students. words that sound the same but are All the spelling questions look the same. They look spelled differently and have different like this one: meanings. A Because of the big test, I 2. Vowel sound mistakes. These questions ? pored over my books. test your knowledge of proper vowel B After college I plan to join the usage. peace corps. 3. Consonant sound mistakes. These C My father usually gives good questions test your knowledge of proper advise. consonant usage. D No mistake In this question, advise should be spelled advice, 4. Ending mistakes. These questions test so the correct answer choice is (C). your ability to spell words with endings such as -ed, -ly, and -ing.

How to Prepare for Spelling 5. Prefix or suffix mistakes. These • Read, read, read! Every time you see the questions test your ability to spell words correct spelling of a word, you are more with prefixes or suffixes. likely to recognize when it is misspelled. 6. No mistakes. These questions test your • Look up every word you misspell or aren’t ability to recognize words that are sure about. Make your own Common spelled correctly. Mistakes list. Copyright © by The McGraw-Hill Companies, Inc. • Pay attention! Some people are naturally great spellers and some are not. But even people who aren’t great spellers can improve their skills if they make improvement a priority. • Check yourself. Every time you write something, go back and check your spelling.

Helpful Hint There is never more than one spelling error in each question. Sometimes, though, all of the underlined words are spelled correctly. Look closely at each word. If you can’t find a mistake, choose No mistake.

Ten Days to the Stanford 9 64 Copyright © by The McGraw-Hill Companies, Inc. the test will alwaysthe test will give you aclueto helpyou findtheincorrectly usedhomophone. Here ishowtoapproachthisquestion: Here isanexampleofahomophonequestion: Look atthissentence,forexample: thewords example, For Homophones are words thatare pronounced alike buthave different andspellings. meanings What isahomophone? They sound thesamebutmeandifferent things. sound They is The word inthissentence thatshouldbeused The word ? 4. 3. 2. 1. fare D C steudrie odue orcl nase C?Ys ti.Eliminate answer (C). itis. Is theunderlinedword usedcorrectly inanswer (C)? Yes, noticed thattheword You may have itis. Is theunderlined word usedcorrectly? Yes, Now lookatanswer (B). market; kind of shouldhave told word you thattheright forthissentence is crafts,” the word In thissentence, itisnot. Is theunderlinedword usedcorrectly? No, Look atanswer (A). A We saw atthefar pigs nwr() Now you canbesure thatanswer (A) isthecorrect choice. answer (D). you caneliminate thesentences, Since you already know thatthere isanerror inoneof B read theremaining choices justto besure. aebtma ifrn hns nti ae however, In thiscase, same butmeandifferent things. used homophones! Eliminate answer (B)andwatch outforcorrectly you. homophone wasincludedto trick sara od u fyura h etne you’ll thattheword incorrectly. notice isused youread thesentence, but if is areal word, No mistake My unclegives usane summer b My sister plansto atthe sellhercrafts His p worries to , too bizarre and , izar hc en wid”i sdi h rn otx.Yu le “sell her Your clue, context. isusedinthewrong which means “weird,” , bizarre r r two e e y . e d e r oohns ic ahudrie odi ato asentence, Since each underlinedword of ispart are homophones. preyed . upon hismind. ecie oehn htsessrne epase A,and Keep answer (A), somethingthatseemsstrange. describes c d ot is ahomophone—thewords es about hislife. fair 65 . The two words— Thetwo . fare preyed and preyed fair sue rpry This is usedproperly. —are homophones. and bazaar prayed .A bazaar sound the is a Vowel Sounds It is important to know when vowels have long sounds and when they have short sounds. Here are some examples of long vowel sounds and short vowel sounds: • Long “a” in atrium ➞ Short “a” in advance • Long “e” in enough ➞ Short “e” in enter • Long “i” in idol ➞ Short “i” in inference • Long “o” in orient ➞ Short “o” in other • Long “u” in university ➞ Short “u” in under Some combinations of vowels make different sounds than you might expect. Here are some examples of words with unusual vowel combinations:

rheu•ma•tism \r 'm -t z' m\

ae•on \'n'\

aer•i•al \âr' -l\

phar•aoh \fâr' \ It is also important to remember that sometimes vowels make no sounds at all. For example: mote \m t\

e•rase \ -r s'\ Here is an example of a vowel sound question: A Is the spot on my dress noticable? B Because I have so many heroes, I can’t ? Copyright © by The McGraw-Hill Companies, Inc. choose just one. C I am genuinely concerned about the crisis at the zoo. D No mistake

Here is how to approach this question: 1. Look at answer (A). Is noticable spelled correctly? No! It should be noticeable.Watch out for words with a silent “e”—they can be tricky! Keep answer (A).

2. Remember to read the other answers just to be sure. Look at answer (B). Is heroes spelled correctly? Yes, it is. Eliminate answer (B).

3. Look at answer (C). Is genuinely spelled correctly? Yes, it is. Eliminate answer (C).

4. Since you already know that there is an error in one of the sentences, you can get rid of answer (D). Now you can be sure that answer (A) is the correct choice.

Ten Days to the Stanford 9 66 Copyright © by The McGraw-Hill Companies, Inc. oeie digasfi ilcag h a osnn ons For example: change thewaySometimes addingasuffixwill aconsonant sounds. Here ishowtoapproachthisquestion: Here isanexampleofaconsonantsoundquestion: words containing hard andsoftconsonant sounds: examples of Here are some Some consonant soundschange dependingontheword appear. inwhich they Consonant Sounds words containing silentconsonants: Here are someexamples of toIt remember isalsoimportant thatsometimesconsonants canbesilent. For example: consonants canmake three different sounds. Sometimes acombination of ? 1. • • • ad“c in Hard “sc” in Hard “g” in Hard “c” B C D A o,btti odi ple orcl.Gtrdo answer (A). of Getrid but thisword isspelled correctly. you, to lookalittle strange It might andisincludedhere you. to distract thedoubleletters, of Is Look atanswer (A). I have o My picture wasaccidentally o out. No mistake the yearbook. This seemsto bearegular o co mag co ma pto char change pneu yacht chap • • op op • • gi candy maine gross c • • • scam ic ac cassio • er • • • mat er er \yät\ cian • mj k\ j' \m • c nj\ \ch ter on • • a ate ➞ • • ➞ ➞ ic tion nal t' n'\ 'm \t m- h n\ sh' -j \m \k \sh \k \ ot“”in Soft “g” n-mt ot“”in Soft “c” l ot“c in Soft “sc” y r occurrence \k ' p - been asked to help - r\ k-t ' p rn -r - ' rt -r p ' ' '\ germ c k\ -r spacious cur '\ spelled correctly? Yes, it is. This is a tricky word Thisisatricky because itis. spelled correctly? Yes, scenic mitt s n\ 'sh 67 r e e nc d e from . 2. Now look at answer (B). Is occassionally spelled correctly? No, it isn’t! Correctly spelled, the word should be “occasionally.”Do you see how answer (A) was trying to distract you? Keep answer (B), and watch out for these tricks!

3. Look at answer (C). Is omitted spelled correctly? Yes, it is. This answer is also meant to be a distractor. Get rid of answer (C). Now you can be sure that (B) is the correct answer.

Endings Many common spelling errors are caused by improperly added word endings. There are quite a few questions that test your knowledge of word endings on the SAT-9. Look at these Endings rules. If you don’t understand any of the examples, ask your teacher for help.

Drop the “e” when the word ends in “e” and you add: -ing, -ion, -ive drape ➞ draping innovate ➞ innovative elevate ➞ elevation create ➞ creating

Don’t drop the “e” when you add -d, -ty locate ➞ located entire ➞ entirety

Additional “e” rules: -ly, -ment, -able exclusive ➞ exclusively erase ➞ erasable lovable ➞ lovably charge ➞ chargeable whole ➞ wholly trace ➞ traceable state ➞ statement abate ➞ abatement argue ➞ argument Copyright © by The McGraw-Hill Companies, Inc.

When a word ends in a consonant, don’t add another consonant before -ed, -ity, -ly, -ment, -ize, -ious, -ous, -ation, -ance valid ➞ validity empower ➞ empowerment anger ➞ angered mountain ➞ mountainous frivolous ➞ frivolously flirt ➞ flirtation uproar ➞ uproarious immortal ➞ immortalize perform ➞ performance watch ➞ watched

Exceptions to the “double consonant” rule: -ed, -ing, concur ➞ concurred strut ➞ strutting fit ➞ fitted run ➞ running

TenTen DaysDays toto thethe StanfordStanford 99 68 Copyright © by The McGraw-Hill Companies, Inc. oecmol iseldwrswt rfxs Somecommonlymisspelledwordswithsuffixes: Some commonlymisspelledwordswithprefixes: words thatare often misspelled. of spellings listsshow Thefollowing thecorrect that test your to prefix spell ability andsuffixwords ontheSAT-9. besomequestions There will wordsSpelling thatcontain prefixes or suffixes canbealittleconfusing. Prefix orSuffix Try thisoneonyourown: Pluralization—ending in“s”vs.“es” Change “y”to“i”whenyouadd-ous,-ness,-hood,-cation,-ly, -cal,ormakeplural Drop lastconsonantwhenyouadd-cy ? e esals oyrefractory gravelly -ory -ly believable salutation -able -tion forcible -ible relevance indigence -ance -ence re- reestablish in- inanimate ex- excruciating dis- dissect un- unnamed im- immeasurable mis- misshape D A tomato hoof fallacy president B C likely mortify lovely manly No mistake Sarah toldSarah funny st Sal visits hissister o Sal visits vacation. Mystery novels are my fa ➞ ➞ ➞ ➞ ➞ ➞ ➞ ovsflat hooves ieiodgeography likelihood oeiesindignity loveliness alns extraordinary manliness ➞ alcossappy fallacious oaosjustice tomatoes otfcto flurry mortification rsdnymalevolent presidency o c r casio y s about her v nal o r it l y es . . ➞ 69 ➞ ➞ flats ➞ sappily flurries ➞ justices ➞ ➞ indignities geographical ➞ malevolency oscontinuous -ous extraordinarily Ten Daystothe Stanford9 Here is an example of a suffix question: A I plan to succeed at what ever I do. ? B My brother has a resistence to change. Helpful Hint C I’ve searched the house thoroughly, but I can’t find my shoes. Did you know that the D No mistake word “misspelled” is often misspelled? Many people Here is how to approach this question: don’t realize that there is a double “s.” 1. Look at answer (A). Is succeed spelled correctly? Yes, it is. This word may look a little strange because of the double letters, but it is spelled correctly. Eliminate answer (A).

2. Look at answer (B). Is the word resistence spelled correctly? No, it isn’t! Correctly spelled, the word should be “resistance.”Watch out for wrong answer choices that confuse “a” and “e.” Keep answer (B).

3. Even though you think you’ve found the correct answer, you should always read the other answers just to be sure. Now look at answer (C). Is the word thoroughly spelled correctly? Yes, it is. Eliminate answer (C).

4. Now you can be sure that (B) is the correct answer.

No Mistake It is important to keep in mind that sometimes all three of the underlined words will be spelled correctly. When you find one of these questions, you should choose the No mistake option. These questions often try to trick you by including words that look strange but are spelled correctly. For example: Copyright © by The McGraw-Hill Companies, Inc. luncheon guerrilla rhythm chaperone receipt colonel martyr counterfeit gauge exaggerate maneuver phenomenon

Can you answer this one on your own? A They were assiduous in following the ? directions. B The dish was covered with microorganisms. C The prophet saw into the future. D No mistake

Ten Days to the Stanford 9 70 Copyright © by The McGraw-Hill Companies, Inc. Tips toRememberforSpelling Common SpellingErors Confusing consonants. Dropping oraddingadoubleletter. wthn I and“E.” Switching “I” Dropping asilentletter. Confusing vowels. spelled correctly anddonotinclude the “e.” underlined word is compare itto asimilarword. you aren’t sure aboutaword, If Sound Error mistakes reading thecomplete without sentences. sentences. words read onlytheunderlined the andignore ontime, you short are running If wordsRead theunderlined all before choosing ananswer. remember to choose you don’t seeanerror, If words theunderlined all correctly. are sometimes spelled inaquestion Remember, He isalways w The money wasabiginse The money I wase Throw intheincine thepaper She tried to fie She tried were d They I wasd We need o need We The ideaisdifficultto c ng umf r ose xig Do this only if you are very short on time, because it will bedifficult to spot becauseitwill you on time, are short very Do thisonlyif ound e cie For example: e d n hinning g in thebook. n v to breathe. e For example: e d an injury. For example: d . For example: hideing Errors ➞ ➞ . I wasd They were d They o ➞ n nt va thinkabout , He isalways w i ➞ ➞ ➞ y v e . r umbf . at We need o need We For example: ➞ I wase She tried to f She tried ➞ e r The ideaisdifficultto c . The money wasabiginc The money 71 e ound ➞ c e ng i Throw intheincine thepaper v gliding e r e xy hining osse d d . . g e e ig No mistake d n , n in thebook. chiding to breathe. . an injury. , siding . o n e v nt e . These similarwords These are . y i v . e . o xml,i the if For example, r at o r . Language How to Prepare for Language Language has three parts: • Read, read, read! Remember—the more you 1. Mechanics. These questions measure read, the more you learn! your ability to recognize and correct mistakes in capitalization, punctuation, • Pay attention to punctuation. Notice how and word usage. commas, colons, and semicolons are used, and practice using them properly in your 2. Sentence Structure and Clarity. In this own writing. section, you are expected to recognize and correct awkward wording and • When you read, pay attention to how things incomplete sentences. are worded. Notice how colorful wording adds excitement to what you read. 3. Style, Context, and Organization. This section tests your ability to combine • Use a thesaurus to help you choose colorful sentences, use colorful language, add words. Practice using colorful language important information, and eliminate when you write and avoid using the same unncessary information. words repeatedly. • Have someone you trust read your writing and help you make it clearer. Ask them to look for mistakes in organization, unnecessary information, run-on sentences, and sentence fragments. Language Part 1: Mechanics Each Mechanics question includes a sentence in which several words are underlined. Your job is to decide whether the underlined words contain a mistake in word usage, punctuation, or capitalization. Copyright © by The McGraw-Hill Companies, Inc. If there is a mistake, choose the answer choice that shows the correct way of writing the underlined words. Keep in mind that sometimes the underlined portion is correct the way it is written.

Each Mechanics question will look like this:

The apples in Janets’ backyard always ? taste good. A apples in Janets’ backyards B apples in Janets’s backyard C apples in Janet’s backyard D Correct as is

Ten Days to the Stanford 9 72 Copyright © by The McGraw-Hill Companies, Inc. • • • • • Capitalization Tips Here ishowtoapproachthisquestion: For example: andholidays. months, Capitalize days, For example: come they before aperson’s when Capitalize titles name. For example: asentence. aquote within Capitalize thefirstletter of For example: names. Capitalize place words of thatare part For example: andlanguages. nationalities, places, people, Capitalize nounssuch proper asnamesof 4. 3. 2. 1. 6. 5. okda o n ad “WhyI looked atTom don’t you pick thatup?” andsaid, New Silver YellowstoneYork Lake Lincoln Avenue, City, National Park, Vietnamese Madagascar, North Carolina, Mark Twain, correct The answer is(C). error anddoesnotintroducepunctuation any othererrors. corrects the “apples inJanet’s backyard” Choice (C), Now you have oneanswer left. to theendof Thisanswer choice addsanextra wrongly “s” Now lookatanswer (B). Eliminate answer (A). plural. changes theword andwrongly (A) repeats error, thepunctuation Answer choice Eliminate choices thatrepeat theerror. Now lookattheanswer choices. Think abouthow you would correct themistake. Janets’ you have might noticed thatthename In theabove sentence, or noerror inthesentence. Remember thatthere iseitheroneerror Read thesentence anddecidewhattheerror is. Cover theanswer choices donotconfuse sothey you. rdy oebr FlagDay November, Friday, Cousin Jethrodie President Abraham Lincoln, Senator Ethel Zimmerman, Eliminate answer choice (B). proper nouns begin with acapitalletter.proper with nounsbegin All the word isan “s.” thelastletter of A word anapostrophe if shouldonlyendwith okdsrnet o.Why? Because looked to you. strange 73 Janets’ has anapostrophe place. inthewrong backyard into the Janet . Here’s another example:

There are three different vegetables on the ? menu tonight, spinach, peas, and broccoli. A tonight, spinach peas, and broccoli B tonight: spinach peas and broccoli C tonight: spinach, peas, and broccoli D Correct as is

Here is how you should approach this question: 1. Cover your answer choices.

2. Read the sentence and decide what the error is. In the above sentence, you may notice an error in punctuation. The phrase “tonight, spinach, peas, and broccoli” is difficult to understand because it contains a list of items that is not preceded by a colon.

3. Think about how you would correct the mistake.

4. Now look at the answer choices. Eliminate the answer choices that repeat the mistake. Look at answer (A). This sentence improperly uses a comma instead of a colon to set off the list. Eliminate answer (A).

5. Since you know that the sample sentence is not correct as it is, you can eliminate answer (D) also.

6. Look at answer (B). Does the phrase “tonight: spinach peas and broccoli” use the proper punctuation? No, it doesn’t. This sentence correctly replaces the first comma with a colon, but then omits some necessary commas. Get rid of answer (B).

7. Look at answer (C). Does this sentence correct the punctuation error? Yes,it does. A colon is used to separate an independent clause from a list in a sentence, and commas separate Copyright © by The McGraw-Hill Companies, Inc. the items in the list. The correct answer is (C).

"Cut out all these exclamation points. An exclamation point is like laughing at your own joke." —F. Scott Fitzgerald

Ten Days to the Stanford 9 74 Copyright © by The McGraw-Hill Companies, Inc. nti ae theword In thiscase, lrl Let’s lookatanotherone: plural. The word • • • • • • Mechanics Tips helping verb Properly written, the sentence should read: thesentence Properly written, doesn’t make any sense. Never usetwo words negative inonesentence. Watch outforindefinite pronouns! For example: theverb shouldalsobeplural. thesubjectisplural, if By thesametoken, should alsobesingular. The subjectandverb must always inthesentence. agree pronoun caseto use. beharder foryou itmight to decidewhich there ismore thanonesubjectinthesentence, If Always make inthecorrect case. sure thatpronouns are written The word Remember thatpossessive pronouns never have apostrophes. All is sapua eas trfr osvrlpol.Thismeansthehelpingverb people. is aplural itrefers toseveral because her’s He isn Is his thebeach ball oritis she, He, They ar They A He isn themar All of M S The mountainbike ishe The mountainbike ishe I am I splashedwater athim must also besingular.must also he l ing l fthesauce is of pahdwtra e (subjectcase) splashed water atme. is incorrect because itactuallymeans because is incorrect is.”“her oe.(singular) bored. and he ’ ’ t t all e making an making no oe.(plural) bored. ssnua eas trfr ooepr ftesnwc.Thismeansthatthe thesandwich. of itrefers toonepart is singularbecause Here’s an example of a pronoun written in the wrong case: inthewrong apronoun written Here’s anexample of r Properly written, the sentence shouldread: thesentence Properly written, e raced home. raced oe.(singular) bored. playing soccer.playing rpigoto my sandwich. outof dripping y sense. ? (possessive case) sense. (object case) . rs r’ s . . aesr hyarewt h eb For example: Make theverb. sure with agree they 75 eesa xml fadoublenegative: Here’s anexample of aig“h onanbk shri”just Saying mountainbike is” isher “The ftesbeti iglr theverb thesubjectissingular, If For example: For example: are must also be must also A good way to tell whether something is written in the proper case is to separate the two subjects into their own sentences like this: Ming raced home. Her raced home. As you’ve probably noticed, the word her is the wrong case to use. Here’s how you would correct it: She raced home. Now you can put it all together: Ming and she raced home.

• Watch out for words that are commonly confused. Here are some examples: Sit and Set Sit means to be seated, as in “to sit still.” Set means to place an object somewhere, as in “Set it on the coaster.” There, Their, and They’re There refers to the location of something, as in “It’s in there.” Their is a pronoun that shows possession, as in “It’s their yard.” They’re means “they are,” as in “They’re not interested.” The apostrophe replaces the missing letter “a.” Lay and Lie Lay is only used when referring to an object, as in “Lay that towel over there.” Lie is a verb that does not directly refer to an object, as in “Sam should lie down.” Copyright © by The McGraw-Hill Companies, Inc. To, Too, and Two To is a preposition as in “to the party.” Too means also or very, as in “I want to go, too” or “You are being too loud.” Two means the number 2.

Try this one on your own: The two most difficult subjects math Here’s a Hint Always use commas ? and science were canceled today. to set off phrases A subjects, math and science, were that describe a noun. B subjects math and science, were C subjects, math and science were D Correct as is

Ten Days to the Stanford 9 76 Copyright © by The McGraw-Hill Companies, Inc. sample sentence iscorrect the way itis. Keep inmindthatsometimes the decide how to fixit. itdoes, if sentence contains amistake and, the Your goalisto determine if andnot redundant. easyto understand, correct, grammatically questionstest your toSentence sentences construct ability andClarity Structure thatare Language Part2:SentenceStructure andClarity Punctuation Tips sentence.) For example: itisacomplete (Aclauseisindependent if Use asemicolon to separate two independentclauses. s oo eoeals.For example: Use acolon before alist. For example: Use apostrophes to show possession. conjunctions are:conjunctions theword above, In thesentence Use commas to: s utto ak oofe ilge For example: Use to quotationmarks offsetdialogue. e f words thatexplainapreceding noun. Set off Separate itemsSeparate inaseries. example: For are They followed by acommon andamainclause. contain asubjectandverb. subordinate clauses. Set off Although he was bigger than me, I beat himatbasketball. Ibeat hewasbiggerthanme, Although ysse ie itn n upe;I oee,prefer lizards. however, I, My sister likes kittens andpuppies; iad iet a h olwn od:fis rshpes rces andbeetles. crickets, grasshoppers, flies, Lizards like foods: to eatthefollowing here?” “Will you pleasegetthatthingoutof She yelled, My lizard crawled undermy sister’s bedroom door. sheseesmy screams when pet. lizards, of isafraid who My sister, ylzr soag,gen andyellow. green, My lizard isorange, after , as , because For example: although Subordinate asubordinate with conjunction clausesbegin and , before is a subordinate conjunction. Some other subordinate Someother is asubordinate conjunction. , since 77 , until For example: , and whatever. Each question in this section will look like this:

Warm and sunny. It was a perfect ? day for a wedding. A Warm and sunny, it was a perfect day for a wedding. B It was warm and sunny, it was a perfect day for a wedding. C Warm and sunny it was a perfect day for a wedding. D Correct as is

Here is how to approach this question: 1. Cover the answer choices.

2. Read the sentences and decide what the error is. Remember that there is either one or no error in the sentence. In the sample above, you may notice that a period separates two incomplete thoughts. An incomplete sentence is called a sentence fragment. You should always avoid sentence fragments.

3. Think about how you would correct the mistake.

4. Now read the answer choices. Answer choice (A) combines the two sentence fragments into one “Style and structure are the coherent sentence. There appear to be no mistakes in this sentence, so keep answer (A). Remember essence of a book; great that even though you think this sentence is ideas are hogwash.” correct, you should always read all of the answer —Vladimir Nabokov choices. Copyright © by The McGraw-Hill Companies, Inc. 5. Since you already know that the sample is not correct as it is, you can eliminate answer (D).

6. Look at answer choice (B). This answer choice incorrectly rewords the sentence in an awkward manner. Get rid of answer (B).

7. Answer choice (C) is also awkward because it doesn’t have the comma that is needed after the phrase “warm and sunny.” Eliminate answer (C).

8. Now you can be sure that (A) is the correct answer.

Ten Days to the Stanford 9 78 Copyright © by The McGraw-Hill Companies, Inc. Here ishowtoapproachthisquestion: Let’s anotherquestion: try ? 3. 2. 1. 7. 6. 5. 4. u-nsnec.Gtrdo answer (B). of rid Get sentence. run-on Thisanswer jumblesthewords around to create anotherawkward Read answer choice (B). Eliminate answer choice (A). worded. probably hadahard timeunderstandinganswer choice (A)becauseitisalsoawkwardly You Now lookattheanswer choices andeliminate theanswers thatrepeat themistake. answer (D). you caneliminate Since you already know thatthesamplesentence isnotcorrect asitis, Think abouthow you would correct themistake. a run-on. you may notice thatthesamplesentence is In thiscase, error ornoerror inthesentence. Remember thatthere iseitherone Read thesamplesentence anddecidewhattheerror is. Cover theanswer choices. D C B A is (C). Thecorrect answer itis. Is clearly? thesentence written Yes, Go onto answer choice (C). Correct asis andIagree. beach ismore fun, Most peoplethinkthe prefer thebeach. butthey enjoyThey goingto thepool, andIagree. think ismore fun, which mostpeople preferthey thebeach, Even enjoy they going to though thepool, agree. andI most peoplethinkismore fun, which preferThey thebeach to thepool, agree. andI thinkismore fun, people most which prefer thebeach, they but enjoyThey goingto thepool, 79 Always make sure Try this one on your own: Here’s a Hint that all of the verb She bought the car to drive to tenses agree in a ? work, take weekend trips, and sentence. running errands. A She bought the car to drive to work, take weekend trips, and ran errands. B She bought the car for driving to work, take weekend trips, and running errands. C She bought the car to drive to work, take weekend trips, and run errands. D Correct as is There is one type of This one is tricky. Give it a try: Here’s a Hint error that is very The book was so good, I read it all difficult to catch ? at once in one sitting. because it is a language problem, A The book I read was so good, I read it all at once in one sitting. not a grammar problem. Can you B The book was so good, I read it in one see a problem in sitting. this example? C The book being so good, I read it in one sitting all at once. D Correct as is

Sentence Structure and Clarity Tips Copyright © by The McGraw-Hill Companies, Inc. • Avoid run-on sentences. Sometimes a long thought can be stated more clearly when you add a period, comma, semicolon, or a conjunction. Here is an example of a run-on sentence:

Stefan is a very good gymnast he has to practice constantly.

Did you notice how hard that sentence was to understand? Here is how you would fix it with a period:

Stefan is very good gymnast. He has to practice constantly.

Here is how you would fix it with a comma and conjunction:

Stefan is very good gymnast, but he has to practice constantly.

Here is how you would fix it with a semicolon:

Stefan is a very good gymnast; he has to practice constantly.

Ten Days to the Stanford 9 80 Copyright © by The McGraw-Hill Companies, Inc. Read theparagraphbelow, andanswerthequestionsthat follow: andorganization. content, andanswer questionsaboutitsstyle, paragraph read ashort you beasked will to In thissection, andusecolorful information language. essential andunnecessary questionstest andOrganization your to combine ability sentences aswell asrecognize Content, Style, Language Part3:Style,Content,andOrganization • • • • •

fyou day hadto to walkupthe stairsevery gethome? if abuilding floorof Could onthethirty-first you living imagine could much. not be usedforvery to build huge f structures People have beenable possible. theelevator thatmadeskyscrapers itwastheinvention of advance, More there thanany would benoNew othertechnological YorkWithout skyline. City elevators,

orha.Your awkward sentences. spoken language canhelpyou your head. “hear” Always thinkabouthow you could reword asentence to improve itsclarity. Remember thatrepetitive clutter wording to asentence. addsunnecessary same form. verbs must inthe all be In any series, Make sure sentences thatall have structure. parallel incomplete canbecombined thoughts to make onecomplete sentence. Watch sentences thatdon’t outforfragmented acomplete form thought. Here it: ishowyouwouldcorrect Avoid comma splices.

Use aperiod,commaandconjunction,semicolontofixthisone:

the answers. thatcontained paper mostof Idiscovered apiece of my textbook, In thebackof the answers. itcontained mostof paper, Idiscovered apiece of my textbook, In thebackof

➟ football when hewasyoung. when football My dadwantsmeto beonthefootball team heplayed ➟

eei neapeo acomma splice: Here isanexample of

o ➟

r alo

ng t ➟

ime ➟

Paragraph 1 Paragraph u ihu nes a orahtetp such structures aneasyway but without to reach thetop, , ➟

81 ➟

Sometimes two ➟

Try “saying” it in it Try “saying” ➟

➟ ➟ Which of the following is the most interesting way to write the underlined ? phrase in the paragraph above? “Style and structure are A since a long time ago the essence of a book; B for many years great ideas are hogwash.” C since the time of the pyramids —Vladimir Nabokov D for some time before now

Here is how to approach this question: 1. Skim the paragraph quickly.

2. Read the first question. Every question will ask you to change or add to the paragraph. The above question asks you to choose the most interesting way to state the underlined words in the paragraph.

3. Before you look at the answer choices, think about how you would improve the underlined phrase.

4. Now read answer (A). Does it make sense? No, it doesn’t. This answer changes the underlined wording into an awkward phrase. Eliminate answer (A).

5. Answer (B) makes sense, but it isn’t any more interesting than the underlined phrase. Get rid of answer (B).

6. Now read answer (C). This answer not only makes sense, but it adds a bit of style to the sentence as well. Answer (C) is probably the correct choice, but you should read on in case the last answer choice is better.

7. Answer (D) is uninteresting and poorly worded. Eliminate answer (D).

8. Because answer (C) is the most interestingly worded answer choice, it is the correct answer. Copyright © by The McGraw-Hill Companies, Inc.

Let’s try the next question:

Which of the following sentences does not fit ? in the paragraph? A The first skyscrapers were built about one hundred years ago. B The Empire State Building is eighty-six stories tall. C Buildings that are hundreds of feet tall are now common. D The invention of the automobile made long-distance travel easier.

Ten Days to the Stanford 9 82 Copyright © by The McGraw-Hill Companies, Inc. • • • Paragraph Syle,Content,andOrganizationTips Here ishowtoapproachthisquestion: motn ato goodwriting. of part important languageisan Colorful better. oneiscertainly thesecond isnotincorrect, sentence While thefirst usingfigurative language: written isit?Here isthesentence aboveisn’tThe sentence interesting, very sentence:following Notice how figurative languageisusedto make asentence more interesting. are combinedlike this: they when flowbetter sentences two These Read example: thefollowing inasentence. phrases You conjunctions. canalsocombine with thoughts words transition areSome other like flowintoeachother this: they read when better The abovesentences Read example: thefollowing relationships between sentences. Use to make your transitions sentences flow smoothly. 4. 3. 2. 1. 5. neto fteeeao,ntteivnino h uooie Thecorrect answer is(D). theautomobile. nottheinvention of theelevator, invention of sentence This focusesonthe doesnotfitbecausetheentire paragraph into theparagraph. Remember—the correct bethesentence answer will thatdoes Now read answer (D). answer (C). Eliminate itdoes. Doesthissentence fitinto theparagraph? Yes, Now read answer (C). Eliminate answer (B). Answer choice (B)could alsobeaddedto thisparagraph. answer choice (A). Eliminate itdoes. Doesthissentence fitinto theparagraph? Yes, Read answer choice (A). sentence thatdoes questionasksyou This to choose a again. Read thequestionandskimparagraph The wind produced green, ocean-like ripples in the long prairie grasses. ocean-like inthelongprairie ripples produced green, The wind sway. grasses madethe longprairie The wind hungry. butIwasstill I ate theentire pizza, hungry. Iwasstill I ate theentire pizza. to them. heisallergic However, Jermaine loves strawberries. He to them. isallergic Jermaine loves strawberries. contain thewords except not lhuh hrfr,meanwhile, therefore, although, belong intheparagraph. and not . These questions canbetricky! questions These . 83 Conjunctions are words thatconnect Transitions are words thatshow Be especially careful on questions that careful onquestions especially Be and except . Read the not fit Study Skills

The Study Skills section of the SAT-9 Here are two types of questions that tests your ability to find information in you will be expected to answer: such places as reference books, newspapers, and library catalogs. 1. General. These questions ask you to find information in reference books like a dictionary, a thesaurus, and Books in Print. How to Prepare for 2. Specific. These questions will give you a sample Study Skills page from a reference book and expect you to Familiarize yourself answer questions based on the information it contains. with the uses of each of the following types of reference materials: • A dictionary gives the pronunciations and meanings of words. • A thesaurus is used for finding synonyms, or words with similar meanings. • An atlas contains maps. • Encyclopedias contain articles in alphabetical order about different things from a wide variety of subjects. • An almanac contains general data, including numbers and statistics. It is not as in-depth as an encyclopedia. • Newspapers contain up-to-date information about current events.

• A telephone directory contains telephone numbers and addresses. Copyright © by The McGraw-Hill Companies, Inc. • Books in Print contains information about all of the books currently being printed and sold. • The Readers’ Guide to Periodical Literature is used for finding recent magazine articles about different subjects.

Tips for Becoming Familiar with Reference Resources • Visit a library to practice finding information in the sources listed above. • Learn the purpose of each part of a dictionary entry. Practice writing a sample dictionary entry and then look up the word to see how close you came. • Practice finding books by using a library catalog. This will help you on questions that refer to an entry in a library catalog. • Learn how to use a book’s index and table of contents. Practice looking up specific topics in the index.

Ten Days to the Stanford 9 84 Copyright © by The McGraw-Hill Companies, Inc. question. Try thisoneonyourown: Here isanothergeneralStudySkills Here ishowtoapproachthisquestion: Here isanexampleofthiskindquestion: in reference books. are questionsaboutfindinginformation The firstseveral questionsintheStudy section general Skills General StudySkillsQuestions ? ? 4. 3. 2. 1. hsoe Now you canbesure that(B)isthecorrect answer. this one. Does theword Eliminate answer (C). one. word Doesthe you ontime.)Lookatanswer are (C). short (You canskipthisstep if make sure. read theotheranswer choices to youAlthough thinkanswer (B)isthecorrect answer, Compare theword Look atanswer (B). You seethat will Compare each letter in Look attheword inanswer (A). D C B A this page? would foundon be thefollowing Which of dictionary. a Look attheseguidewords from apageof D C B A you would use— text, line of a to hyphenate aword attheendof To way findtheproper h w ud od,you know onthispageinthe dictionary. itbelongs the two guidewords, Becausetheword intheanswer choice fitsalphabeticallybetween answer choicewith (A). sandal salivate salve sandbar an almanac an encyclopedia a thesaurus a dictionary salivate salute –sample pero hspg ntedcinr?N,itappearsonapagebefore this No, appear onthispageinthedictionary? sandal sandbar pero hspg ntedcinr?N,itappearsonapageafter No, appear onthispageinthedictionary? appears Here’s aHint after 85 hspg ntedcinr.Gtrdo answer (A). of Getrid this pageinthedictionary. salve with thewordswith A An An A thesaurus dictionary statistics. including numbersand words. about awidevarietyofsubjects. and syllables. about wordpronunciations encyclopedia almanac sandbar salute lists synonymsof gives youinformation contains generaldata with each letter in with contains articles and sample as you did sample . Specific Study Skills Questions Most of the Study Skills section is divided into groups of questions about a particular reference or resource. The resource might be a page from a dictionary, a library catalog entry, a book index, or another tool you might use to study. Here is an example from the Practice Test:

Use this Table of Contents to answer the following questions:

CONTENTS INDEX acid, definition of Chapter 1 Arrhenius 79 Atoms ...... 3 Brönsted-Lowry 80 Chapter 2 Lewis 82 Gases ...... 36 atomic radius 11 Chapter 3 Bohr 6 Liquids ...... 61 Boyle’s law 45 Chapter 4 boiling point 47, 68 Acids ...... 78 buffers 100 concentration, Chapter 5 molality 66 Nuclear Decay ...... 105 molarity 65 Charles’ law 40 Dalton’s law 58 In which chapter would you find electron 4, 14Ð22 information about protons? freezing point 69 ? A chapter 1 Graham’s law 41 B chapter 2 Hund’s law 21 C chapter 3 mass number 10 neutron 8Ð11 Copyright © by The McGraw-Hill Companies, Inc. D chapter 4 nuclear decay, types of, Here is how to approach this question: alpha decay 111 beta decay 108 1. Look at answer (A). Look in the index 114 to determine which pages discuss periodic table 6, 13, 28 protons. The index shows that proton 5Ð9 protons are discussed on pages 5–9. quantum theory 30 Now look in the table of contents to valence 20Ð21 find out which chapter contains pages 5–9. Could you find information about protons in chapter 1? Yes! Answer (A) is correct.

2. If you have time, double check the remaining answers to make sure you have chosen correctly. Look at answer (B). Could you find information about protons in chapter 2? No. Eliminate answer (B).

Ten Days to the Stanford 9 86 Copyright © by The McGraw-Hill Companies, Inc. Here ishowtoapproachthisquestion: thenextquestion: Now try ? 5. 4. 3. 4. 3. 2. 1. tdent ae2 spr ftecatraotaos Thecorrect answer is(D). thechapter aboutatoms. it doesn’t! Page of 21ispart DoesHund’s Law onpage21appearinthechapter No, aboutgases? Look atanswer (D). Eliminate answer (C). it does. DoesDalton’s Law onpage58appearinthechapter aboutgases? Look atanswerYes, (C). Eliminate answer (B). itis. about gases? Yes, thechapter of Is page40part contents to determine which chapter itis in. at thetableof Law intheindex andthenlook Charles’ Lookupthepagenumber of Look atanswer (B). you caneliminate answer (A). chapter aboutgases, Since Boyle’s Law appearsinthe contents to findoutwhich chapter page45appearsin. Now lookatthetableof The index shows thatBoyle’s Law isdiscussedonpage45. LookupBoyle’s Law intheindex to pageitappearson. findoutwhat Look atanswer (A). D C B A gases? with laws thefollowing does Which of Now you canbesure thatanswer (A)isthecorrect choice. answer (D). of rid Get Could you aboutprotons findinformation inchapter 4?No. Look atanswer (D). answer (C). of rid Get Could you aboutprotons findinformation inchapter 3?No. Look atanswer (C). Hund’s law Dalton’s law law Charles’ Boyle’s law not 87 deal Tips to Remember for Study Skills Questions

Prepare for these questions by learning to use all the different types of information sources before you take the SAT-9!

Eliminate the answers that you know are wrong first.

Always refer back to the sample information entries with each new answer choice. Don’t rely on your memory!

Be sure to read each question slowly and carefully. Careless reading will result in mistakes!

If you don’t know the answer to a question, guess and move on. Copyright © by The McGraw-Hill Companies, Inc.

Ten Days to the Stanford 9 88 Copyright © by The McGraw-Hill Companies, Inc. project. about where thestudentshouldgoto orhow findinformation thestudentshouldorganize the questions You then beasked acouple will of or areport. aletter, astory, bewriting student might the For instance, asituationfaced by astudent. You of description bepresented ashort will with Study Skills • • • following: dothe thetest, of the LanguageSAportion your teacher tells you thatyou betaking will If How toPrepareforLanguageSA way you would inreal life. section requires you to usetheseskillsthesame LanguageSA The aswell. and Languagesections Study Skills, you donottake theSpelling, test if the You onlytake will theLanguageSAsectionof andLanguagesections. Study Skills, the Spelling, The LanguageSAquestionstest as thesameskills The LanguageSATest iuaielnug nalo your homework assignments. figurative of languageinall thisbook. of section theLanguage Review usingthem. and practice thisbook. of section theStudySkills Review thewords thatyou misspellonhomework assignments. Look upallof thisbook. of section theSpelling Review rcieuigpoe rma,pntain and punctuation, Practice usingproper grammar, 89 Learn about all of the information source theinformation books aboutallof Learn “To even or write 3. 2. 1. Language SAquestions: There arethreetypesof —George Orwell science but an art.” speak English is not a correctly spelledwords. to recognizeability bothmisspelled and Spelling. grammar. and style writing your knowledge of Skills. Language study tools. to usereferenceability booksandother Study Skills. These questionstest your These questionstest your These questionstest Here is an example from the Practice Test:

A rough draft of Simon’s report is below. Use the rough draft to answer questions 7 and 8. The Life of Samuel Taylor Coleridge Samuel Taylor Coleridge was one of the greatest poets in the (1) English language. He wrote during the Romantic period of English (2) literature, which took place in the late 1700s and early 1800s.

His most famous poems are The Rime of the Ancient Mariner and (3) Kubla Khan.

Samuel Taylor Coleridge was born in England in the year 1772. (4) He was an imaginative child who kept to himself. Rather than to (5) (6) play with other children, books was what he preferred. He began (7) writing poetry as a teenager. A sonnet is a poem that is (8) fourteen lines long. He went to college in Cambridge. (9)

Which of these would be the least important ? item for Simon to try to include in his

report? Copyright © by The McGraw-Hill Companies, Inc. F The place where Coleridge lived G The titles of some of Coleridge’s poems H The foods Coleridge liked to eat J The time when Coleridge lived

Here is how to approach this question: 1. Read the passage and question slowly and carefully.

2. Look at answer (A). Should Simon include information about where Coleridge lived? Yes. Describing where someone grew up is important information to include in a biography. Remember—the correct answer will be the one that is least important to include in a report. These questions can be confusing, so be careful! Get rid of answer (A).

3. Look at answer (B). Should Simon include the names of some of Coleridge’s poems? Yes—this information is important. Eliminate answer (B).

Ten Days to the Stanford 9 90 Copyright © by The McGraw-Hill Companies, Inc. Here ishowtoapproachthisquestion: and askyou to correct thegrammar. underlineaphrase will they beaboutgrammar; questionswill Other or which sentences don’t belong. may they askyou which sentences canbecombined beaboutstyle; Somequestionswill be improved. thestudent’s project beasked andyou questionsabouthow will itcan You begiven aversion will of Style andGrammar ? 6. 5. 4. 7. 6. 5. 4. 3. 2. 1. Eliminate answer (A). thissentence isawkward. Is thissentence correctly? No, written Now lookatanswer (A). Think abouthow you would improve it. You’ve probably noticed thatthissentence doesn’tRead sentence 6. make much sense. Cover your answer choices don’t sothey confuse you. Now you canbesure that(C)isthebestanswer. (D). Eliminate answer lived? Yes—this to includeinabiography. isimportant information aboutthetimethatColeridge ShouldSimon includeinformation Look atanswer (D). Keep answer (C). not really important. itis beinteresting itmight information, Although itisn’t. Coleridge’s favorite foods?No, forSimon to Is about includeinformation itimportant Now lookatanswer (C). Now you canbesure thatanswer (B)isthecorrect choice. answer (D). of Getrid Thisanswer isalsoworded awkwardly. Look atanswer (D). Eliminate answer (C). thissentence isawkward. Like answer (A), No. Is thissentence correctly? written Look atanswer (C). the answer choices justto besure. you shouldalways readEven allof you though thinkyou have foundthecorrect answer, Keep answer (B). itis. Is this sentence correctly? written Yes, Look atanswer (B). D C B A What sentence isthebestway 6? to write Best asis otherchildren. playing with of To read booksiswhathepreferred instead otherchildren.with He preferred reading booksto playing children waswhathepreferred. other Books more thanplaying with 91 Here is another question based on the same passage. Try to answer it on your own.

Which sentence does not fit in the report? ? A 5 B 8 C 2 D 3 Spelling You will be given a sentence with three underlined words and you will be asked if one of them is misspelled.

Here is an example:

The subject of the artical was an arrogant person. No mistake. ? FG H I

Here is how to approach this question: 1. Read the sentence slowly and carefully. As you read, decide if any of the underlined words contain an error.

2. Look at answer (A). Is the word subject spelled correctly? Yes, it is. Get rid of answer (A).

3. Look at answer (B). Is artical spelled correctly? No, it isn’t! Correctly spelled, the word should be article. Keep answer (B).

4. Even though you think you know the right answer, you should read the rest of the answer choices just to be sure. Is the word arrogant spelled correctly? Yes, it is. Eliminate answer (C). Copyright © by The McGraw-Hill Companies, Inc.

5. Since you know that the sentence contains a mistake, you can get rid of answer (D) .

6. Now you can be sure that (B) is the correct answer.

Tips to Remember for Language SA

The questions on this part of the test are mostly just a review of the material in the Spelling, Study Skills, and Language sections.

Always eliminate the answers that you know are wrong first.

Remember to read all of the answer choices.

Read the passages, questions, and answer choices slowly and carefully.

Ten Days to the Stanford 9 92 Copyright © by The McGraw-Hill Companies, Inc. Copyright © by The McGraw-Hill Companies, Inc. Copyright © by The McGraw-Hill Companies, Inc. Copyright © by The McGraw-Hill Companies, Inc.